Wednesday, December 25, 2019

National Institute Of Mental Health Essay - 727 Words

National Institute of Mental Health: Panic disorder The National Institute of Mental Health (NIMH, 2015) mission is to transform the understanding and treatment of mental illnesses (NIMH, 2015). NIMH try to accomplish this though clinical research, so they can prevent and cure people. Their goals are to promoting the discovery of the brain, so they can find the cure for the mental disorder, and developing new intervention that ingrate the need and support for people with mental illnesses (NIMH, 2015). The NIMH does research on several mental disorders like depression, eating disorder, schizophrenia, social phobia and panic disorder. Panic disorder is a serious mental disorder, which is a sudden attack of fear (WebMD, 2015-2016). People have those attacks without warning and can happen when a person is not in danger. Symptoms include sudden and repeated attacks of fear, racing heart rate, sweating, trouble breathing, dizziness and chest pain, stomach pain and avoidance of place wher e attach have occurred in the past (Panic Disorder, n.d.). Panic disorder affects about 6 million adults, it’s twice as common in women as men, and begin in late adolescence or early adulthood. (Panic Disorder, n.d.). This paper will describe the history of NIMH and current trends and the future of panic disorder. The NIMH has a very long history and has changed some much over the years. On July 31946, President Truman signed the National Mental Health Act, which called for the establishmentShow MoreRelatedThe National Institute of Mental Health661 Words   |  3 PagesThe National Institute of Mental Health has dedicated itself to mental health issues across the board since 1946 when President Harry Truman put the National Mental Health Act in place. This government program has developed to become a huge organization in concurrence ingwith the issues of mental illnesses.The National Institute of Mental health also known as NIMH Has an excellent mission statement,which is The mission of the National Institute of Mental Health (NIMH) is to transform the understandingRead MoreThe National Institute Of Mental Health1200 Words   |  5 PagesIn 2013, the National Institute of Mental Health (NIMH) broadly launched the Research Domain Criteria project (RDoC). The purpose of RDoC is to provide a working framework to advance dimensional approaches of classifying psychopathology on the basis of observable behaviors and neurobiological measures (National Institute of Mental Health, 2013). Reflecting on the last thirty years, incredible progress has been made in understanding brain-behavior relationships in numerous mental disorders. The continuedRead MoreThe National Institute Of Mental Health1690 Words   |  7 PagesAccording to a study in the journal of Pediatric Dentistry, parents of children with autism define their children’s oral health as fair or poor. Children with Autism Spectrum Disorder have difficulties adjusting with the dental scenery, due mainly to their inability to communicate and socialize their thoughts and feelings. Dental office visits can be challenging to the child with autism and therefor the parent as well. It is our job as a dental professional to help make these office visits asRead MoreNational Institute Of Mental Health Services893 Words   |  4 PagesAbuse and Mental Health Services Administration (SAMHSA) promotes prevention and treatment of mental disorders through Center for Mental Health Services (CMHS). CMHS is charged with making policy-level decisions for any modifications made to the existing vocational rehabilitation methods. National Institute of Mental Health (NIMH) is a research organization which promotes mental health through basic and clinical research. It works on the areas of prevention, recovery and cure of mental illnessesRead MoreEating Disorders And The National Institute Of Mental Health1597 Words   |  7 PagesDiana Schimenti Julie Lartz College Writing 20 April 2016 Eating Disorders According to The National Institute of Mental Health (NIMH) website, eating disorders are actually serious and often fatal illnesses causing serve disturbances to a person’s eating behaviors. People with eating disorders often have obsessions with food, their body weight, and their shape. There is, however, a difference between an eating disorder and a diet. It is important to know the difference. Eating disorders are aRead MoreAn Analysis Of The National Institute Of Mental Health Clinics894 Words   |  4 PagesAs noted in Diagram 1, many primary care physicians are referring new patients to mental health clinics at alarming rates with the result being long wait times for the patient, as the hospitals struggle to keep up with the referrals. The National Institute of Mental Health (NIMH), it has noted that there are an approximately 57.7 million people per year between the ages of 18 and older have a diagnosable mental health disorder (NIMA, 2016). Many people are subjecte d to prolonged and unnecessary sufferingRead MoreAccording To The National Institute Of Mental Health, Social1315 Words   |  6 PagesAccording to the National Institute of Mental Health, social anxiety is a mental disorder with a lifetime prevalence of 12%. It is characterized by a persistent, intense and chronic fear of being scrutinized by others when engaging in, as well as a fear of, social interactions. In this form of anxiety, individuals are afraid of saying or doing something that will embarrass or humiliate them and have unrealistic appraisals of the negative consequences of social encounters. There are many factors thatRead MoreChildren With Bipolar Disorder : The National Institute Of Mental Health1590 Words   |  7 PagesSabrina Peters Research Essay Professor Wissman November 5, 2015 Children with Bipolar Disorder The National Institute of Mental Health has defined bipolar disorder as a serious mental illness that is marked by extreme changes in mood that range between two states: manic and depressive. Bipolar disorder, also known as manic depression, is classified as a mood disorder. This disorder goes beyond the typical ups and downs. Bipolar disorder is becoming more prevalent and a much more seriousRead MoreDepression And Bipolar Disorder : The National Institute Of Mental Health Essay1358 Words   |  6 PagesDisorder Whitney Keeton Mr. John Davis Jr., MFA, M. Ed. English Composition 1 - ENC 1101 Monday, Tuesday, and Thursday 6-10 PM Keiser University â€Æ' Depression and Bipolar Disorder The National Institute of Mental Health (NIMH) has stated that depression is a common but serious mood disorder. Depression is also a form of mental illness; that have many different types. Some of the types of depression are as follows: †¢ Persistent depressive disorder †¢ Perinatal depression †¢ Psychotic depression †¢ SeasonalRead MoreChildren With Bipolar Disorder : The National Institute Of Mental Health1752 Words   |  8 PagesSabrina Peters Research Essay Child Adolescent Dev. Professor Wissman November 5, 2015 Children with Bipolar Disorder The National Institute of Mental Health has defined bipolar disorder as a serious mental illness that is marked by extreme changes in mood that range between two states: manic and depressive. Bipolar disorder, also known as manic depression, is classified as a mood disorder. This disorder goes beyond the typical ups and downs. Bipolar disorder is becoming more prevalent

Tuesday, December 17, 2019

Comparing the Loss of Innocence in Cullens Incident and...

Loss of Innocence in Cullens Incident and Naylor’s Mommy, What Does Nigger Mean? Unfortunately, a question that many African Americans have to ask in childhood is Mommy, what does nigger mean?, and the answer to this question depicts the racism that still thrives in America (345). Both Gloria Naylor’s Mommy, What Does Nigger Mean? and Countee Cullens Incident demonstrate how a word like nigger destroys a child’s innocence and initiates the child into a world of racism. Though the situations provoking the racial slur differ, the word nigger has the same effect on the young Naylor and the child in Cullen’s poem. A racist society devours the white children’s innocence, and, consequently, the white children embody†¦show more content†¦Unlike Naylor, Cullen never identifies the sex of the black child in his poem. Perhaps Cullen renders the child in his poem genderless to give the child a more universal standing; perhaps he leaves the child genderless in order to focus on the more important fact that the child, whether male or fema le, sees no difference between him- or herself and the other boy until the â€Å"Baltimorean boy calls him/her nigger (3). Both Cullen and Naylor add color to the description of the children with this single racial epithet. The white children’s use of the word nigger establishes a distinction between them and the black children in Naylor and Cullen’s works which embodies the essence of racism. This distinction forces the young Naylor and the child in Cullen’s poem to see beyond their innocence and to see themselves and their world in new colors: black and white. Both Naylor and Cullen touch on an important issue by noting that the first incident of racism for the black children, occurring when the white children call them nigger, takes place between two children. The fact that the white boys call the young Naylor and the child in Cullen’s poem nigger at such a young age reflects the unfortunate truth that America teaches color boundaries and racism at a young age. The word nigger does not exist in white children’s vocabulary at birth. Rather, their parents and community teach them the word and pass down the legacy of racism as if it is an

Monday, December 9, 2019

Osteoarthritis and Cartilage

Question: Describe the strategies that, you as a nurse, can implement to reduce the severity and impact of the disease for the individual and their family? Answer: Osteoarthritis(OA) is also known asdegenerative joint disease, degenerative arthritisoror osteoarthrosis, is a type of abnormalities which involves joints degradation, as well as subchondral bone and articular cartilage.The purpose of the assignment is to develop an instrument so as to assess the difficulties faced by the people in their daily lives due to the Osteoarthritis of the Knee (Biological measurement of osteoarthritis, 1993). The symptoms of the disease and the strategies that I will take as a nurse are highlighted in the assignment. Pathophysiology Osteoarthritis (OA) is considered a progressive joint disease and causes disability. It can occur to people of any age especially who are 45 years and above are more likely to be affected by the disease. The Arthritis Foundation says that that more than 27 million people living in the United States are have the chances of having osteoarthritis, where the knee is more likely area to be affected area. It was also seen that the women are more prone to osteoarthritis than the men. The disease is characterized by the loss of the arterial cartilage, subchondral sclerosis of the bone, formation of osteophyte and cysts. The remodeling of the subchondral bone plays an important role in the OA Pathophysiology (14 Evaluating pain in osteoarthritis, 2005). It is often called Osteoprotegerin (OPG). As the draining down of the hyaline cartilage progresses, hypertrophic changes can occur in the synovial tissue and also the underlying bone with which can form sclerosis and osteophyte. This often lea ds to narrowing of space of the joints and joint surface become rough and irregular which ultimately causes pain, and swelling in the joint deformity (Biological measurement of osteoarthritis, 1993). Symptoms The symptoms of the disease are severe pain in the knee, with dysfunction and discomfort in the area. Thus it can be said that the pain is the initial symptom of the disease. Osteoarthritis causes pain in the knee area and reduces the motion of the person affecting the quality of life. As time passes, the condition of the knee detoriate. There is no definite treatment for the disease, and it is managed by the controlling the pain and preserving the function of the affected area (Knee taping reduces symptoms associated with osteoarthritis, 2003). The control of the life style, changes in the diet, exercises, and the use of medication and orthosis can control the pain and improve the condition to some extent. In extreme stages it can be seen that the only possible way out is knee replacement. Treatment and care by nurses It can be said that the task of the nurses is to control the pain of the knee and the joint and to promote the quality of the life of the people who are suffering from the disease. As a nurse I can help the people by guiding them and letting them know the signs and symptoms of the disease. They can be guided with education so that they can take care of themselves in case of need by doing simple exercises (Ringdahl, Erika and Pandit, 2011). The nurses instruct the patients reduce their weight and use devices that can be used to support their walking like a walker or a walking stick. I as a nurse can encourage the people in active participation to control the progress of the disease. The nurses can teach the patients exercises and the use of various aid which will help the people to keep their knees in the state of motion (gutierrez, 2013). Moreover physiotherapy can help the people to reduce the level of pain and improvements can be seen. The nurses can follow up the medication prescr ibed by the doctors and monitor the improvements and also can implement new exercises if it is necessary. References 14 Evaluating pain in osteoarthritis. (2005).Osteoarthritis and Cartilage, 13, pp.S5-S5. Biological measurement of osteoarthritis. (1993).Osteoarthritis and Cartilage, 1(1), pp.21-21. gutierrez, L. (2013). Improvement of symptoms in the generalized osteoarthritis use of zoledronic acid zola study.Osteoarthritis and Cartilage, 21, p.S293. Knee taping reduces symptoms associated with osteoarthritis. (2003).BMJ, 327(7407), pp.0-c-0. Ringdahl, Erika, and Pandit, S. (2011). Treatment of Knee Osteoarthritis.American Family Physician, 83(11), pp.1287-1292.

Sunday, December 1, 2019

Psychological Foundation of Human Resource Development

Introduction The effective management of human resource is the means through which organizations can succeed in continued growth and financial stability. The discipline of human resource development has been considered to be based on the theories of not only economic and systems but also on the psychological theories of the surrounding the human resource department.Advertising We will write a custom essay sample on Psychological Foundation of Human Resource Development specifically for you for only $16.05 $11/page Learn More However, there has not been enough exploration on the implications associated with the economic theories on the human resource development theories. In this context, human resource development entails the process which focuses on bringing out the best of human expertise with the sole purpose of improving and maximizing the performance of n organization. The dominating theoretical understanding of human resource development is the int egration of all theories that may seem to be significant in an organization’s operations as well as promoting result oriented functionality. This can only be achieved by human resource development through creation of a much friendlier working environment by improving the health of workers from a psychological and social perspective (Swanson, 2008, p. 1). This paper examines the need for the human resource development department to keep checks on the well-being of the employees by discussing the possible factors that may affect the well-being of the workers with much emphasis on stress audits. Assessment of the psychosocial workplace With the increasing competition in markets, human resources get under a lot of pressure to produce high quality products and services in order to maintain the organizations’ position in the market. Such pressure may be prolonged enough to cause strain on the employees. This could affect the physical, psychological and behavioral capabilitie s of the employees. Physical strains affect the employees’ to perform bodily duties due to muscle aches, headaches, and other discomforts in the body. This has been confirmed in various researches which show that some disorders such as musculoskeletal are a result of workplace stress. Psychological strain mostly affects the employees ability to perform effectively hence may cause a reduction in productivity. They are indicated by depression and anxiety both of which relatively causes job dissatisfaction. On the other hand, behavioral strains involve the unusual and most likely unethical behaviors on the part of the employees at the workplace. They include addiction to alcohol or medicines, uncontrollable consumption of food and most importantly, unprofessional interaction with other workers or even superiors.Advertising Looking for essay on business economics? Let's see if we can help you! Get your first paper with 15% OFF Learn More Consequently, when employee s are strained the organizations get affected too. For instance, absenteeism may result from employee stress especially where the strain is a result of poor working environment. Losses incurred during compensation to stressed or injured employees are likely to result (Thomas, butts, Vandernberg, Dejoy and Wilson 2005, p. 4790. Similarly, compensation on workers and expenses on health schemes increase. This is where the human resource comes in to ensure that the psychosocial lives of the employees are maintained steady by creating favorable working environment which would then result to healthier production. However, many organizations do not pay much attention to workers’ stressful conditions but rather leave it to the employees to deal with it. Organizations should therefore work more on employee stress and this entails getting involved in individual level stress management and not just organizing stress management courses and that is it. This entails the participation of th e organization in identifying and implementing effective stress busters. The most effective method that is appropriate for human resource development is the use of stress audit which is a systematic tool used to measure stressors as well as stress busters in a workplace environment. Stress audits are effective in identifying performance related stressors that would not have been identified in the traditional assessments of human resource development. Despite the fact that stress audits are meant to address organization related stress, there is also the need to examine the most recognized stressors especially those that promote the health of the employees. This entails the assessment of organizational factors that affect the physical, social and psychological being of the employees (Quill, 1999, 19). However, the most effective factors to assess are the range of work load on the employees, the behavior of the workers’ supervisor, social support availed by the organization to t he employees and the control method used by the organization on the workers. These factors are significant to all organizational settings and have most impacts on the employee performance and behavior.Advertising We will write a custom essay sample on Psychological Foundation of Human Resource Development specifically for you for only $16.05 $11/page Learn More The aspect of job control entails the extent to which workers are allowed to have control on their duties. According to research, worker who have low control over their jobs are less healthy than those with high control. It is obviously stressful for anyone to be controlled in all aspects of work. Therefore, addressing stress associated with low job control should not, by any means, involve the implementation of more control from the superiors. However, increasing the job control should be carefully undertaken to avoid giving the employees certain responsibilities that they view as legitimately t heir superiors’. The most effective measures to solving this problem would be re-scheduling tasks, work policies and duty procedures as well as including breaks for resting. These physical arrangements help workers feel more at ease with their work. Work overload, on the other hand, refers to a situation in which workers have too much work and a limited time to do the work. This may be referred to as quantitative work load with a qualitative work load involving a situation where workers have complex duties which may cause strain on them. Generally, work overload has serious effects on the health of workers including heart related conditions. The effect of work load on employees may increase due to the need of workers to balance between work and life and this is precisely what results to signs of strain. This could be solved by utilizing organizational resources as much as possible as well as giving employees time to recover from such work-related pressures (Hood 2001, p. 165) . Similarly, lack of support especially from the managers is one of the greatest contributors to employee stress. Social support such as being attentive to workers and offering them ideas and counseling services are some of the important ways of giving social support to employees. Instrumental support is as important as social support especially through helping workers in aspects such as advising, coaching, job training or by mentoring employees. Social support is not only important in promoting health in organizations, but at all levels of individuals’ lives.Advertising Looking for essay on business economics? Let's see if we can help you! Get your first paper with 15% OFF Learn More Psychological health is better supported through social support from family, friends, work mates and job managers as well. On the other hand, social support is significant in the work place as it helps transfer the skills attained from the training session to the real working environment. Social support can be created by emphasizing on team work and cooperation between all stakeholders in an organization. This prevents employees from strain that results due to tension and emotional tiredness. Work supervisors have great influence on the employee’s well being since they determine how workers feel about them and how well or bad they perceive them. Inconsiderate supervisors may cause psychological strain on workers by being too hard on them and pressuring them especially on work load related matters (Kahnweiler, 1991, p. 347). This is because supervisors are in charge of job delegation and planning and it is precisely how they do this that affects the employee performance and he alth. Supervisor self reflection is one of the significant ways in which originations can foster in efforts of trying to avoid compromise on the psychological health of employees. Conclusion As much as all organizations want to succeed especially in maintaining their market positions in the competitive world of business, they should also understand the need to maintain good health for their employees because they are the foundation to the organizations’ performance. Since most workers get stressed due to work related issues, it is the duty of human resource development to ensure that favorable working conditions are created for the employees in order to promote healthy performance (Biggie and Hunt 1992, p. 623). Psychological fitness is the most important tool in work performance and as long as workers are not provided with environments that promote psychological health, their performance can never be as efficient as the organizations want it to be. All in all, the workers sh ould work alongside their superiors towards ensuring maximum production in organizations by devoting themselves to their work but in a healthy way. However, one question which arises from this paper is â€Å"who is to blame for organization’s poor performance both in production and in the market?† Reference List Biggie, M., and Hunt, M. (1962). Psychological foundations of education: An introduction to human development and learning. Universal digital library: Harper Brothers. Hood, P. (2001). Human resource development: motivation and movement. Management in education 15 (19). Kahnweiler, W (1991, November). HRD and empowerment. Training and development. New York: EBSCO. Quill, W. (1999, summer). The psychology of human resource development; a consequent of radical cultural change in America. Journal of business and psychology, 13 (4). Swanson, R. (2008). Economic foundation of human resource development: advancing the theory and practice of the discipline. Thomas, W ., Butts, M., Vandernberg, R., Dejoy, D., and Wilson, M,. (2005). Effects of management communication, opportunity for learning and work schedule flexibility on organizational commitment. Journal of vocational behavior, 68 (2006) 474-489. This essay on Psychological Foundation of Human Resource Development was written and submitted by user Evan Walton to help you with your own studies. You are free to use it for research and reference purposes in order to write your own paper; however, you must cite it accordingly. You can donate your paper here.

Tuesday, November 26, 2019

Signode Industries, Inc

Signode Industries, Inc Recommendations Mr. Gary Reed should increase the price of Steel Strapping Consumables by 3.6% and tie the salesperson's compensation to gross margin instead of sales revenues. He should Maintain the loyalty of existing customers in National and Large segment by providing value added services like providing custom sizes, grades, tools and machines. He should also focus on stimulating further volume growth by taking actions to convert nonusers into users, to increase use frequency among current users, or to expand into untapped or underdeveloped markets.Pricing Decision Pricing decision in this case involve an inherent conflict between (1) the need to win customers and (2) the need to maintain/increase profit margins to satisfy the firm's financial need to generate as much capital as possible for further expansion. The complete analysis for arriving the pricing decision was shown in the Exhibit 1. The revenue generated by steel consumables is $133 millions. Its existing margins percen tage is 36.5.Bentley's winged "B" badge and bonnet (hood) ornam...The 6.8% increase in material cost resulted in $4.8 million burden on Signode. Signode either have to pass on this burden to customer in the form of price increase or absorb the burden by reducing its margins. Signodes past experience showed that the introduction of price increases and substitute products like plastic-strapping materials resulted in 10% drop in market share. So it is assumed that the price increases would result in 10% reduction in revenues. Gross revenues are calculated for the three options: ‚Â · Increasing price ‚Â · Keeping the price constant ‚Â · Price Flex It is evident from Exhibit 1 that "Increasing price" option provides greater gross margin compared to "Keeping the price constant" or "Price Flex" options. So Gary Reed should increase the prices to maximize the gross margins for Signode.Sales personnel Compensation Current sales personnel compensation is tied to the...

Saturday, November 23, 2019

Erwin Schrödinger and the Schrödingers Cat Experiment

Erwin Schrà ¶dinger and the Schrà ¶dinger's Cat Experiment Erwin Rudolf Josef Alexander Schrà ¶dinger (born on August 12, 1887 in Vienna, Austria) was a physicist who conducted groundbreaking work in quantum mechanics, a field which studies how energy and matter behave at very small length scales. In 1926, Schrà ¶dinger developed an equation that predicted where an electron would be located in an atom. In 1933, he received a Nobel Prize for this work, along with physicist Paul Dirac. Fast Facts: Erwin Schrà ¶dinger Full Name: Erwin Rudolf Josef Alexander Schrà ¶dingerKnown For: Physicist who developed the Schrà ¶dinger equation, which signified a great stride for quantum mechanics. Also developed the thought experiment known as â€Å"Schrà ¶dinger’s Cat.†Born: August 12, 1887 in Vienna, AustriaDied: January 4, 1961 in Vienna, AustriaParents: Rudolf and Georgine Schrà ¶dingerSpouse: Annemarie BertelChild: Ruth Georgie Erica (b. 1934)Education: University of ViennaAwards: with quantum theorist, Paul A.M. Dirac awarded 1933 Nobel Prize in Physics.Publications: What Is Life? (1944), Nature and the Greeks  (1954), and My View of the World  (1961). Schrà ¶dinger may be more popularly known for â€Å"Schrà ¶dinger’s Cat,† a thought experiment he devised in 1935 to illustrate problems with a common interpretation of quantum mechanics. Early Years and Education Schrà ¶dinger was the only child of Rudolf Schrà ¶dinger – a linoleum and oilcloth factory worker who had inherited the business from his father – and Georgine, the daughter of a chemistry professor of Rudolf’s. Schrà ¶dinger’s upbringing emphasized cultural appreciation and advancement in both science and art. Schrà ¶dinger was educated by a tutor and by his father at home. At the age of 11, he entered the Akademische Gymnasium in Vienna, a school focused on classical education and training in physics and mathematics. There, he enjoyed learning classical languages, foreign poetry, physics, and mathematics, but hated memorizing what he termed â€Å"incidental† dates and facts. Schrà ¶dinger continued his studies at the University of Vienna, which he entered in 1906. He earned his PhD in physics in 1910 under the guidance of Friedrich Hasenà ¶hrl, whom Schrà ¶dinger considered to be one of his greatest intellectual influences. Hasenà ¶hrl was a student of physicist Ludwig Boltzmann, a renowned scientist known for his work in statistical mechanics. After Schrà ¶dinger received his PhD, he worked as an assistant to Franz Exner, another student of Boltzmann’s, until being drafted at the beginning of World War I. Career Beginnings In 1920, Schrà ¶dinger married Annemarie Bertel and moved with her to Jena, Germany to work as the assistant of physicist Max Wien. From there, he became faculty at a number of universities over a short period of time, first becoming a junior professor in Stuttgart, then a full professor at Breslau, before joining the University of Zurich as a professor in 1921. Schrà ¶dinger’s subsequent six years at Zurich were some of the most important in his professional career. At the University of Zurich, Schrà ¶dinger developed a theory that significantly advanced the understanding of quantum physics. He published a series of papers – about one per month – on wave mechanics. In particular, the first paper, â€Å"Quantization as an Eigenvalue Problem, introduced what would become known as the Schrà ¶dinger equation, now a central part of quantum mechanics. Schrà ¶dinger was awarded the Nobel Prize for this discovery in 1933. Schrà ¶dinger’s Equation Schrà ¶dingers equation mathematically described the wavelike nature of systems governed by quantum mechanics. With this equation, Schrà ¶dinger provided a way to not only study the behaviors of these systems, but also to predict how they behave. Though there was much initial debate about what Schrà ¶dinger’s equation meant, scientists eventually interpreted it as the probability of finding an electron somewhere in space. Schrà ¶dinger’s Cat Schrà ¶dinger formulated this thought experiment in response to the Copenhagen interpretation of quantum mechanics, which states that a particle described by quantum mechanics exists in all possible states at the same time, until it is observed and is forced to choose one state. Heres an example: consider a light that can light up either red or green. When we are not looking at the light, we assume that it is both red and green. However, when we look at it, the light must force itself to be either red or green, and that is the color we see. Schrà ¶dinger did not agree with this interpretation. He created a different thought experiment, called Schrà ¶dingers Cat, to illustrate his concerns. In the Schrà ¶dingers Cat experiment, a cat is placed inside a sealed box with a radioactive substance and a poisonous gas. If the radioactive substance decayed, it would release the gas and kill the cat. If not, the cat would be alive. Because we do not know whether the cat is alive or dead, it is considered both alive and dead until someone opens the box and sees for themselves what the state of the cat is. Thus, simply by looking into the box, someone has magically made the cat alive or dead even though that is impossible. Influences on Schrà ¶dinger’s Work Schrà ¶dinger did not leave much information about the scientists and theories that influenced his own work. However, historians have pieced together some of those influences, which include: Louis de Broglie, a physicist, introduced the concept of â€Å"matter waves. Schrà ¶dinger had read de Broglie’s thesis as well as a footnote written by Albert Einstein, which spoke positively about de Broglie’s work. Schrà ¶dinger was also asked to discuss de Broglie’s work at a seminar hosted by both the University of Zurich and another university, ETH Zurich.Boltzmann. Schrà ¶dinger considered Boltzmann’s statistical approach to physics his â€Å"first love in science,† and much of his scientific education followed in the tradition of Boltzmann.Schrà ¶dinger’s previous work on the quantum theory of gases, which studied gases from the perspective of quantum mechanics. In one of his papers on the quantum theory of gases, â€Å"On Einstein’s Gas Theory,† Schrà ¶dinger applied de Broglie’s theory on matter waves to help explain the behavior of gases. Later Career and Death In 1933, the same year he won the Nobel Prize, Schrà ¶dinger resigned his professorship at the University of Berlin, which he had joined in 1927, in response to the Nazi takeover of Germany and the dismissal of Jewish scientists. He subsequently moved to England, and later to Austria. However, in 1938, Hitler invaded Austria, forcing Schrà ¶dinger, now an established anti-Nazi, to flee to Rome. In 1939, Schrà ¶dinger moved to Dublin, Ireland, where he remained until his return to Vienna in 1956. Schrà ¶dinger died of tuberculosis on January 4, 1961 in Vienna, the city where he was born. He was 73 years old. Sources Fischer E. We are all aspects of one single being: An introduction to Erwin Schrà ¶dinger. Soc Res, 1984; 51(3): 809-835.Heitler W. â€Å"Erwin Schrà ¶dinger, 1887-1961.† Biogr Mem Fellows Royal Soc, 1961; 7: 221-228.Masters B. â€Å"Erwin Schrà ¶dinger’s path to wave mechanics.† Opt Photonics News, 2014; 25(2): 32-39.Moore W. Schrà ¶dinger: Life and thought. Cambridge University Press; 1989.Schrà ¶dinger: Centenary celebration of a polymath. Ed. Clive Kilmister, Cambridge University Press; 1987.Schrà ¶dinger E. â€Å"Quantisierung als Eigenwertproblem, erste Mitteilung.†Ann. Phys., 1926; 79: 361-376.Teresi D. The lone ranger of quantum mechanics. The New York Times website. https://www.nytimes.com/1990/01/07/books/the-lone-ranger-of-quantum-mechanics.html. 1990.

Thursday, November 21, 2019

Adopting a mac system Assignment Example | Topics and Well Written Essays - 3500 words

Adopting a mac system - Assignment Example de a mock website enabling the management to visualise how a website may benefit their business and also for the report to elaborate on any technical issue that may become apparent during or after the transition. The aim of this project is to integrate the knowledge and skills learned through out the HNC/HND Computing course and to deliver a practical and realistic solution to the business related problem that I have chosen. The objectives for the project are to demonstrate my ability in producing a technically sound document that highlights the required details mentioned in my project proposal and to provide an effective and economical solution to the proposed scenario. During the process of compiling the required information it will be necessary to provide a very brief systems analysis of the old system, it is not my intention to do a complete analysis of the system but I shall provide enough information and data on this topic to allow the project to be completed. LM photography is a company which deals with photography who currently specialize in Weddings and special occasions, they also have two in-house studios where they take family and portrait photographs. The company was started in the year 2005 by Lisa Mary Moggach to turn a life long passion into a sustainable, financially viable business. The business took of well in 2006 when two large wedding planning companies approached LM photography and asked them to supply photographers and equipment for 196 weddings over two years. LM photography took the opportunity and drafted in ten freelance photographers and ten assistant to cope with the additional workload. In 2009 the fledgling company are still flourishing but the management have realized that there current IT system does not have the foundations to be continually built on, as the system can be unstable and unreliable when in constant use. The decision was made to employ an IT company who were to propose a new Macintosh based operating system

Tuesday, November 19, 2019

Themes & Corresponding Essay Example | Topics and Well Written Essays - 1500 words

Themes & Corresponding - Essay Example Smith and Gordimer are able to use these female characters to in their literary works to depict the manner in which the society treats them, because of the color of their skin, as well as their racial background. The short story by Gordimer and the poem by Smith all depict the life of a black woman. Altikriti (2011) explains that racialism and ethnicity are vices that people experience on a day to day basis. On most occasions, black people are always discriminated upon, by the whites or the Caucasians. Furthermore, Altikriti (2011) explains that racialism and ethnicity was a subject that was difficult to discuss, mainly because of the emotions that it could bring forth. These two literary pieces of art are about ethnicity and racialism. Both the short story and the poem give a discussion of ethnicity and racialism. For instance, in the short story, there is a love relationship between Thebedi, a black woman, and Paulus, the son of the master of Thebedi. Altikriti (2011) explains that this relationship between Paulus and Thebedi was forbidden because of the racial nature of Thebedi. She was a black woman, and the black’s were not allowed to inter-marry with the whites. The relationship between Thebedi and Paulus emanated from their childhood romance, into adulthood, resulting to the pregnancy of Thebedi, without the knowledge of Paulus who had gone to study. This passage is able to show the love that existed between Thebedi, and Paulus, Gordimer in her 1975 piece of work denotes that, â€Å"In one summer afternoon, when water was flowing in the river, and it was very hot outside, Thebedi wadded as they used to, when they were little children, the dress she wore was able to bunch modestly tucking into her leg s. The school girls that Paulus used to go swimming with at the pools or dams near the neighboring farms had their bikinis. However, the sight of their thighs and bellies had never made Paulus to feel

Sunday, November 17, 2019

Fahrenheit 451 by Ray Bradbury Essay Example for Free

Fahrenheit 451 by Ray Bradbury Essay â€Å"There are worse crimes than burning books. One of them is not reading them† -Ray Bradbury. In the past there were events that affected book writers. People will get together to burn books because they thought it was inappropriate or they were against their literature. Montag is a fireman in a futuristic society who would start fires instead of put them out. After he meets Clarisse a young girl different from all teenagers in that society Montag will find himself doing things he never did before. In Fahrenheit 451 Montag will have a huge change in his life that will make him different from other people. There were people who contributed to that change, people who Montag would never thought of meeting or people he had already knew. Clarisse had Montag analyzing his happiness and questioning himself which caused him to do things he never done before. When Montag and Clarisse were walking Clarisse asked him if he was happy with the life he was living. â€Å"Of course I’m happy. What does she think? I’m not?†. Montag seems really bothered by the question Clarisse asked him. Montag is positive about his happiness, he knows he’s happy with the life he’s living He knows he’s â€Å"happy† but by someone else asking him that kind of question it doesn’t seem like he is. â€Å"He wore his happiness like a mask and the girl had run across the lawn with the mask and there was no way of going to knock on her door and ask for it†. Once Clarisse asked him about his happiness he started thinking about it and he knew he wasn’t happy at all. He thinks that Clarisse opened his eyes about it. Montag also feels like Clarisse was the one who took his happiness by asking him that question. Mildred proved to Montag he wasn’t really by the way she acts towards him. She was the one who woke him up and made him do something so he could have his happiness back. Mildred as a wife should listen to Montag and make him feel better. Instead Mildred would totally ignore him and seem careless every time he would say something. â €Å"I’m hungry Last night he began. Didn’t sleep well. Feel terrible she said. God I’m hungry I can’t figure it Last night he said again. She watched his lips casually. What about last night Don’t you remember†. Mildred doesn’t really listens to Montag He always tries to tell her something but she seems careless about it. He feels like he’s not getting the attention he wants from her. â€Å"â€Å"When did we meet? Where?†Ã¢â‚¬ ¦ â€Å"It doesn’t matter.† She was up, in the bathroom now, and he heard the water running, and the swallowing sound she made. â€Å"No, I guess not,† he said. This demonstrates that all this years of marriage meant nothing to Mildred. By her not remembering how they meet tells that Mildred doesn’t really care about their marriage. Faber encouraged Montag to overcome his fears not by giving him advice but by showing Montag he also feared. Faber was always the type to follow the rules so he wouldn’t get in trouble and face the consequences. Montag didn’t want to be a coward like Faber that’s how he overcame his fears. â€Å"For a little while I’m not afraid. Maybe it’s because I’m doing the right thing at last. Maybe because I’ve done trash thing and don’t want to look the coward to you†. Faber has never made a change in his life because he has always been afraid of the consequences. He tells Montag that he has to face his fears which make him feel like he’s doing the right thing for once. â€Å"â€Å"Mr. Montag you’re looking at a coward. I saw the way things were going a long time back. I said nothing. I’m one of the innocents who could have spoken up and put when no one would listen to the ‘guilty’†¦Ã¢â‚¬  Faber is telling Montag that he never had the courage to stand up in what he strongly believes its right. He categories himself as a person who’s afraid by the consequences. In Fahrenheit 451 by Ray Bradbury, Clarisse, Mildred and Faber had Montag doing things he never did before. They had him questioning, making him realize and encouraging him to get over his fears. He was doing things he never saw himself doing before in his life. Even though those things he do had its bad consequences he never felt that good about himself. Many people won’t confront their fears because they’re scared of the consequences but once people they overcome those fears there will be nobody who will stop them to do what they think it’s right. WORK CITED Bradbury, Ray. Fahrenheit 451. Simon and Shuster paperbacks; New York, 2012

Thursday, November 14, 2019

JavaScript :: essays research papers

Within this report is information on how Java Script is used on the Internet. This report will contain information that will help you learn about Java Script and its uses on the Internet. It mentions what Java Script is, where it originated and what some of its uses are.JavaScript is a loosely typed scripting language that resembles the programming language C. It is designed to be an extension to HTML and is usually included within HTML scripts. Java Script is object-oriented and has block-structuring features. The main feature of Java Script is that it lets you build interactive web pages. Java Script allows you to respond to a user's input in the form of text, buttons selected, or mouse clicks, and can generate a response to the user's computer. Java Script originated from within the labs of Netscape Communications Corporation in 1994. It was invented so a computing language could build web pages and applications more effectively. When it first appeared it was thought that it was a new form of C, but later turned out to be this wonderful new language called JavaScript. There were skeptics that said the language was a dud and would never catch on, but now more people are learning Java Script then ever before.JavaScript is used within HTML tags so that the page setup in already present when the Java Script starts to run. There are many different uses for Java Script on the Net. It can be used to make mail forms that a customer or person trying to input information can use. The program will allow the user to input information, then it will automatically send that info to a designated email address with the click of a button. It can also be used to put a password on a certain web page; a user trying to enter the page must then input the correct commands and passwords to enter the page. Another command added to this code will make it so that if the user enters the wrong password the code will send them to a site of the scripts designers choice. Java Script is also a popular language for making simple interactive games like bridge and peg games. These codes can provide hours of fun! Java Script is used mainly on pages that need to allow users to enter information or choose from options on the page.

Tuesday, November 12, 2019

Organizational Culture: Present Trends Essay

Organizational culture has been defined as â€Å"the set of shared values and norms that control organizational members’ interactions with each other and with suppliers, customers, and other people outside the organization† (Jones, 2004). Just as an organization’s structure can be used to achieve competitive advantage and promote stakeholder interests, an organization’s culture can be used to increase organizational effectiveness. This is because organizational culture controls the way members make decisions, the way they interpret and manage the organization’s environment, what they do with information, and how they behave. Culture thus affects an organization’s competitive position. As culture is discussed in terms of the values and norms that influence its members’ behavior, it usually determines how members of a firm interpret the environment, bond its members to the organization, and give it a competitive advantage. Recent advances that develop organizational theories that deals with culture in organizations have been instituted. These are developing high performance teams, managing organizational identity and managing diversity. Organizational culture exercises a potent form of control over the interactions of organizational members with each other and with outsiders. By supplying people with a toolbox of values, norms, and rules that tell them how to behave, organizational culture is instrumental in determining how they interpret and react to a situation. In developing concepts that enhance organization’s culture would literally translate success and competitive advantage in organizations. High Performance Teams  When working with a group or a company, one should be a team player in order for your tasks to be accomplished successfully. To quote, â€Å"Someone may be great at his or her job, maybe even the best there ever was. But what counts at work is the organization’s success, not personal success. After all, if your organization fails, it does not matter how great you were; you are just as unemployed as everyone else† (Johnson, Kantner & Kikora, 1990). In the work environment, teams materialize to focus on tasks or solve problems that are beyond the capacity of one individual. With this type of set-up that allow creative and innovative juices to flow through the constant sharing of information, people could appropriate division of labor among the members of the team can lead to more effective, more efficient and less stressful workplace. Their high levels of performance with regards to quantity, quality, and timeliness of work results can contribute to their sense of satisfaction, addressing a psychological and motivational need. With incessant exposure to each other, team members and their superiors could ascertain whether they have a team that can continue working together with synergy or their togetherness poses a detrimental effect on their output and interrelationships. In traditional organizational structures, teams were introduced and experimented on to see what works and what does not. While employees have already formed their own social groups for their personal interaction with other employees, management devised ways to re-group them and build teams that would be more productive for the organization. The old hierarchies were replaced with cross-functional structures that were both flat and self-directed. The emergence of the concept of High Performance Teams evolved. To date, organizations and businesses have shifted to this kind of paradigm. They are depicted as flexible but difficult to put together, expensive but worth every cent. To build a high performance team requires a lot of work, time, effort and money. The team leader can serve to be the conduit between the team and the management or other external organizations. Coordination of the team’s activities is also the responsibility of the team leader. Any team leader should be able to ensure that the team maintains the ethical standards of the organization. What’s important is for team members to be consistently coached by management or external agencies hired by management to continually trust, respect and support each other and the organization. Coaching coupled with their guidelines will keep in check their members’ behavior and enhance their decision making skills. Empowerment is a key for the advancement of these skills. To be empowered, the team needs to have information and resources. It also needs the management’s trust that they won’t abuse the information or the resources they are given, which is often curtailed by the guidelines they have set for themselves. This empowerment leads them to become cross-functional. They are then given a wider perspective of the processes and a detailed coverage of the activities that occur and address what needs improvement in the organization. In developing work environments, more corporations are now staunch in their support for diversity. Dealing with diversity in a way that makes it a strength has come to be known as â€Å"managing† diversity. According to Sharon Nelton: Managing diversity meant, and still means, fostering an environment in which workers of all kinds—men, women, white, disabled, homosexual, straight, elderly—can flourish and, given opportunities to reach their full potential and contribute at the highest level, can give top performance to a company (p. 19). When we refer to â€Å"diversity†, this could mean cultural, demographic, organizational or psychological and encompasses ethnicity, religion, gender, age, personality, values, attitudes, occupations, status, or job tenure. By working together in well-supervised teams that include women and men, young and old, minorities and non-minorities, employees can learn how to realize the full potential of diversity. According to Goetsch & Davis (2004), diversity in teamwork can be promoted by applying the following strategies: †¢ Continually assessing circumstances. Is communication among diverse team members positive? Do bias and stereotyping exist among team members? Do minorities and non-minorities with comparable jobs and qualifications earn comparable wages? Factors that might undermine harmonious teamwork should be anticipated, identified, and handled. Giving team members opportunities to learn. Humans naturally tend to distrust people who are different, whether the differences are attributed to gender, culture, age, race, or any other factor. Just working with people who are different can help overcome this unfortunate but natural human tendency. However, it usually takes more than just working together to break down barriers and turn a diverse group of employees into a mutually supportive, complementary team in which the effectiveness of the whole is greater than the sum of its parts. With regards to compensation, there should be an implementation of an appropriate compensation system. In other words, if you want teamwork to work, make it pay. This does not mean that employees are no longer compensated as individuals. Rather, the most successful compensation systems combine both individual and team pay. In Anne Schauber’s study (2001), it found that if a team’s performance is duly rewarded by the organization, a culturally diverse organization â€Å"may be more economical in the long run† and â€Å"will result in better service to a changing clientele†. It enhances the creativity and problem-solving capabilities of the organization† in such a way that the â€Å"previously untapped talent and energy will be focused on achieving organizational goals† (Schauber, 2001). Thus, diversity has become a positive contributing factor to the achievement of the goals of a high performance team. Moreover, De Vries and Manfred (2005) recently used the idea of Zen Buddhism in leadership group coaching to develop high performance teams. De Vries and Manfred (2005) said that Zen Buddhism has as its fundamental purpose the awakening of the mind and the individual attainment of spiritual enlightenment. A Zen teacher is concerned with self-help and helping others with wisdom and compassion. Given this mindset, Zen teachers can be seen as forerunners of leadership coaches. Like Zen teachers, such management coaches provide learning opportunities by giving constructive and balanced feedback. They serve as sparring partners. They help their clients reflect on their own actions. As a way of clarifying and enhancing consciousness, coaching has become the Zen for executives. With executives finally realizing the value of coaching, the coaching market—now a multi-billion-dollar enterprise—is ballooning. Originally carried out by â€Å"one-person bands,† leadership coaching has become a major activity for many large consulting firms. As corporations are constantly seeking methods to improve their own workplace effectiveness and efficiency, individual and group performance had to be measured. Work teams transform to become empowered to make decisions and improve performance; there is also an increased need for accountability. Virtually, all organizations with work teams need a means for measuring their teams’ performance. Indeed, high performance teams coupled with diversity could spell the success of any organization or corporation in our fast changing global environment. Managing Organizational Identity Organizational identity differs, most sharply, from organizational culture because of the prominent role of transference phenomena. The nature of emotional attachments and connectedness, or disconnectedness, is the footing of organizational life and the essence of organizational identity. The centrality of this emotional substructure is especially crucial when there is demand for organizational change and development. Change depends on members’ willingness to assume responsibility for their actions and to depart from the status quo. But this willingness is the result of mutual understanding of shared emotions between superordinates and subordinates, and often among peers in organizations, and is the outcome of their recognition of unconscious expectations and desires. Helping members to become aware of the structure of organizational identity and their place in it is a precondition for freeing them up for organizational change that is strategically sound and productive (Diamond, 1993, p. 7). Ravasi and Schultz (2006) had presented a recent longitudinal study of organizational responses to environmental changes that induce members to question aspects of their organization’s identity. Their findings highlight the role of organizational culture as a source of cues supporting â€Å"sensemaking† action carried out by leaders as they reevaluate their conceptualization of their organization, and as a platform for â€Å"sensegiving† actions aimed at affecting internal perceptions. Ravasi and Schultz (2006) explored organizational responses to environmental changes and shifting external representations that induced members to reflect on their organization’s recent and prospective courses of action and ask themselves, â€Å"What is this organization really about? † Although past research has documented the impact of desired images on organizational responses to environmental changes, they deemed that the influence of organizational culture—and in particular, the influence of its manifestations—on the redefinition of members’ collective self-perceptions. They found organizational culture became the central construct in understanding the evolution of organizational identities in the face of environmental changes, suggesting that collective history, organizational symbols, and consolidated practices provide cues that help members make new sense of what their organization is really about and give that new sense to others. Furthermore, the role of culture in preserving a sense of distinctiveness and continuity as organizational identity is subjected to explicit reevaluation. The findings suggest that the roles external images and organizational culture play in affecting organizational responses to identity threats may be more complementary than the current literature on organizational identity would suggest (Ravasi & Schultz, 2006). Building on evidence from their research, they developed a theoretical framework for understanding how the interplay of construed images and organizational culture shapes changes in institutional claims and shared understandings about the identity of an organization.

Saturday, November 9, 2019

Argumentative Thoreau Essay

Jessica Taylor 13, February 2012 AP English Mrs. Mercer Self-Sufficiency and Individualism Can Harm a Community Henry David Thoreau goes to the woods to live away from duties and to live a life of leisure. He moves far away from any method of communication, such as the post office. He wishes to live independently and self-sufficiently. The quote â€Å"I went to the woods because I wished to live deliberately, to front only the essential facts of life†¦and not, when I came to die, discover that I had not lived. † He summarizes his reasons for living in the woods in this quote.Thoreau wants to live deliberately; to choose his own course and have no one else influence his life. He doesn’t want to get old and look back and realize that there are things in life that he wanted to do, that he has never done. If everyone in a community lived by the virtues of individualism and self-sufficiency, then there would be no community. In a community, everyone contributes somethin g. Individuals in a community work together, in order to solve problems and issues facing the community. Such as, if there has been a terrible natural disaster, communities work together to repair the damage.If the damage is more extensive in some areas than others, then the individuals of that community can work harder to restore it. If everyone in a community lived by the virtues of individualism and self-sufficiency, then in times of crisis or suffering, there would be no sense of community where people helped one another. Also if a community lived by these virtues, then there would be no trust. In order to gain or build trust, there has to be some sort of connection between individuals. In Thoreau’s self-sufficient world, there was no communication.If a person held the virtue of self-sufficiency, this may make other individuals feel offended. People would feel offended because today’s world is all about communication. If an individual blocked out other people and d id not allow them to help him/her, then this might make the other individual feel as if they were not beneficial. Instead of being a community, it would be more of a rivalry between individuals. There would be no trust or companionship. In times when individuals would need each other’s help; there would be no sense of alliance.Everyone would be against one another, because everyone would feel as if each other were untrustworthy. In the United States, we are one giant community; there are many communities within the major one. In times of war or battle, we stand together. Above all, the citizens in the U. S are able to come together to fight the battles when needed. If every citizen in the U. S lived by the virtues of self-sufficiency and individualism then there would be no U. S. in times of war; there would be no army. In an army you must work together to find the best solution to the problem, there has to be teamwork and trust.In a self-sufficient world, everyone would want to do things in their own way, and in return this would cause chaos. A community is a beneficial factor, without it we have nothing. Yes, being self-sufficient can be a helpful virtue, but in the condition of the community it is one virtue that can destroy the community. If there was no sense of an alliance, there would be no way to survive a natural disaster or fatal event. A community can strive and recover from a setback, whereas, if an individual was self-sufficient, then this would be much more difficult.

Thursday, November 7, 2019

Free Essays on Columbus Day

An Extra Day Off Sounds Nice â€Å"It is only radical or currently unfashionable ideas that the texts leave out- it is all ideas, including those of their heroes.† (FitzGerald) USF does not recognize a major holiday. A lot of schools do not recognize this holiday. This upsets and disappoints me. The holiday that I am speaking of is Columbus Day. My barber shop is not open, neither is my favorite liquor store. Why is my school still open? The reason is simply because many people see Christopher Columbus as a tyrant and do not think we should observe this holiday. If we are going to celebrate Presidents Day and Independence Day then why not celebrate Columbus Day. Our country can kill and it is justified, and Columbus killed yet it is some how frowned upon. I believe we need to respect and show thanks to the man that lead to our founding as country. Christopher Columbus set sail for the East Indies in the year 1492. â€Å"Columbus outfitted three pitifully small ships, the Nina, the Pinta, and the Santa Maria, and set forth from Spain.† (Lowen 39) This shows that Columbus did not set off on an easy voyage. It shows that we need to look at the hardships he went through to bring the new world to Spain. Whether or not he killed, raped, and pillaged the people he came across in the new world he still made it there and brought the news back to Spain. That is reason enough to respect and give thanks to him for me. I certainly would not have the guts to set sail into the unknown to find something that I might not reap the full benefits of. Some say, â€Å"Columbus did not even know for certain where he was going. Evidence says he was seeking Japan, India, and Indonesia.† (Loewn 55) This does not matter. Who cares where he thought he was going. He found a new land that had not been discovered and that land is the land where millions reside. People say if Columbus did not bring this news back to Spain someone else would have. That I agree wit... Free Essays on Columbus Day Free Essays on Columbus Day An Extra Day Off Sounds Nice â€Å"It is only radical or currently unfashionable ideas that the texts leave out- it is all ideas, including those of their heroes.† (FitzGerald) USF does not recognize a major holiday. A lot of schools do not recognize this holiday. This upsets and disappoints me. The holiday that I am speaking of is Columbus Day. My barber shop is not open, neither is my favorite liquor store. Why is my school still open? The reason is simply because many people see Christopher Columbus as a tyrant and do not think we should observe this holiday. If we are going to celebrate Presidents Day and Independence Day then why not celebrate Columbus Day. Our country can kill and it is justified, and Columbus killed yet it is some how frowned upon. I believe we need to respect and show thanks to the man that lead to our founding as country. Christopher Columbus set sail for the East Indies in the year 1492. â€Å"Columbus outfitted three pitifully small ships, the Nina, the Pinta, and the Santa Maria, and set forth from Spain.† (Lowen 39) This shows that Columbus did not set off on an easy voyage. It shows that we need to look at the hardships he went through to bring the new world to Spain. Whether or not he killed, raped, and pillaged the people he came across in the new world he still made it there and brought the news back to Spain. That is reason enough to respect and give thanks to him for me. I certainly would not have the guts to set sail into the unknown to find something that I might not reap the full benefits of. Some say, â€Å"Columbus did not even know for certain where he was going. Evidence says he was seeking Japan, India, and Indonesia.† (Loewn 55) This does not matter. Who cares where he thought he was going. He found a new land that had not been discovered and that land is the land where millions reside. People say if Columbus did not bring this news back to Spain someone else would have. That I agree wit...

Tuesday, November 5, 2019

A Level System Supports Positive Behavior

A Level System Supports Positive Behavior A level system for behavior contract is in many ways a sophisticated system for improving and shaping students long term behavior. By establishing levels, much as in a rubric for academic performance, you can shape students behavior by slowly increasing the expectations for meeting each level. This system is particularly good for secondary students, and can help a student in a single class or across classes. Creating a Level System Choosing Behaviors to Monitor Begin by identifying which behaviors will pull the cart of the students behavior. In other words if you successfully identify behaviors that are pivotal for improving a students over all performance and behavior in your class, focus on them. Behaviors need to be explicit and measurable, although data collection is not your primary focus. Still, avoid general, subjective terms like respectful, or attitude. Focus on the behaviors that will eliminate the attitude. Instead of shows respect for peers you need to identify the behavior as Waits to be called on or Waits rather than interrupts peers. You cannot tell your students what to feel. You can tell them what their behavior should look like. Choose 4 or 5 behaviors that will define the levels: i.e. PunctualityConforming to rules.Completing assignments,Participation Some people would include listening but I find that some secondary students who appear to be ignoring the teacher may actually be listening. You can ask for certain kinds of academic behavior that show whether a student has been attending or not. You cannot actually see students listening. Define the Behaviors for Each Level Describe what is excellent, good, or poor punctuality. Excellent may be on time and ready to learn. Good might be on time. And poor would be late or tardy. Determine Consequences for the Students Behavior Positive consequences can be given weekly or daily, depending on the age and maturity of the student or the intensity or inappropriateness of the behavior. For students with grossly inappropriate behavior , or who have a long way to go, you may want to reward performance daily. As a student participates in a behavior support program, over time, you want to thin reinforcement as well as spread it out so that students learn eventually to evaluate their own behavior and reward themselves for appropriate behavior. Consequences can be positive (a reward) or negative (loss of privileges) depending on the number of excellents or the number of poors each student earns. Decide Who will Provide the Reinforcement I would try to get parents to do the reinforcing if at all possible. Secondary students are especially gifted at working teachers against parents or parents against teacher. When you have parents on board, you are more likely to get the cooperation of a student. It also make lessons learned in school generalize to the students home. There is also nothing wrong with double dipping, providing one level of reward at school (i.e. a privilege earned for so many excellents) and another at home (a trip to a preferred restaurant with the family for so many excellents in a week, etc.) Evaluate and Re-evaluate Eventually, your goal is for students to learn to self-evaluate. You want to Fade from supporting the students behavior. You want to achieve these by. Increasing the time you evaluate, from daily to weekly.Raise the level of behavior you want the student to exhibit for each behavior (especially academic behavior. Tools for a Level Behavior System A Contract: Your contract needs to lay out the who, what, where, when, how of your system. Who: The students who will perform the behavior, the parent(s) who will reinforce the appropriate behavior and the teacher(s) who will evaluate the students behavior.What: Behavior you want to see increase. Remember, keep it positive.Where: all classes, or just one where the student is struggling? Do mom and want to continue the plan at home? (include levels for cleaning the room, say, or touching base with parents when out with friends?)When: Daily? Each period? Weekly? Remember to make it often enough to quickly increase the behavior, but understand that you will eventually be thinning reinforcement by spreading the incidence of reinforcement over longer intervals.How: Who is the evaluator? Will you give the student input on evaluation, or will it all be on you? Monitoring Tools: You want to create a tool that will make it easy for you or for general education teachers who may be evaluating students. I offer you models for A contract for a single class with a key.A blank contract for a single class.A week for a self contained program.A week for multiple classes.

Sunday, November 3, 2019

Woodrow Wilson's Fourteen Points Essay Example | Topics and Well Written Essays - 500 words

Woodrow Wilson's Fourteen Points - Essay Example He is particularly renowned for his fourteen points that developed a basis to the end of the war and the post World War I era. This paper explores the success Woodrow’s fourteen points and the points’ achievements. Overview of Woodrow’s fourteen points Woodrow’s fourteen points was a proclamation towards the end of the First World War. Having come just a few months away from the ultimate end of the war and their provisions that aimed at attaining a peaceful world indicate significance of the points in the consequently achieved peace. The fourteen points’ principles can be explored in three categories. The first category, consisting of initial five points, was general and concerned â€Å"international liberalism and free trade† (Rossini and Shugaar, p. 63). The points included a public approach to agreements towards international peace as opposed to private treaties, provision for freedom of navigation across international waters, elimination of trade barriers and equality in international trade, and appeal for disarmament. He also called for fair distribution of colonies (Yale, p. 1). Points that called for resolution of disputes over some territories followed these.

Thursday, October 31, 2019

MMM CO Essay Example | Topics and Well Written Essays - 750 words

MMM CO - Essay Example trust on the organization, increase the performance efficiency, grab the maximum market share and what strategies a company would implement to approach the customers. Internal drivers comprise of situations where the company is influenced to manage its profitability and retain its customers by applying various internal strategies. The internal drivers within an organization are centered to deliver: According to Michael Porter’s book, ‘Competitive Advantage: Creating and Sustaining Superior Performance† published in 1985, the concept of Value Chain was first coined (Michael Porter). According to this concept, value chain comprise of the entire primary and the secondary activities within an organization that would help the firm to strengthen its position in the market as well as to improve its product or service efficiency within the market. At this stage, it is important for MMM & Co. to focus on all the activities of the organization that are related with the receiving and storing of the raw materials especially those products which are used by the company to produce the end products. The better the inbound logistic system of the company would be, the better would be quality of the products and the raw materials. MMM & Co. needs to focus its attention on the operations within the company that initiate from the manufacturing of the products which involves the receiving of the raw materials and end on the final production. Throughout the operations, the company is in dire need to maintain the flow of the operations smooth as well as, to maintain the control on all the steps. Outbound logistics play a key role in getting the finish products to the customers. It revolves around the handling and shipping of the products from one place to another. The smooth flow of outbound logistics plays a crucial role in determining the profitability of the company. MMM & co. needs to focus its attention on this activity. Marketing and sales stand as the corner stone to

Tuesday, October 29, 2019

Essay Example | Topics and Well Written Essays - 1000 words - 26

Essay Example Also, when Gregor has turned into a bug, we find the family to be self-sacrificing as well. Even though it distresses her to do so, Gergor’s sister feels a sense of duty to him, and she makes sure he is fed. When at first he cannot eat his favorite meal, his sister does everything that she can in order to find food that he will eat. She loves him, and out of a sense of duty she tries to feed him and keep him alive, even though he no longer resembles the person that he used o be in any way, shape, or form. The family has to sacrifice some of their own food money so that Gregor can eat, and this shows that they love him, even though they can’t stand to look at him. In Robert Frost’s â€Å"Home Burial,† we see two different conceptions of love through the act of mourning. In the poem, a married couple’s child has died, and the grave is on their property. The husband and wife have very different ways of coping with the loss of the child. The wife, Amy, stands at the window and stares at the grave. She is in a constant state of sorrow, and she can barely do anything because she has been so greatly affected. Because of her love for her child, she has grown distant from her husband, who is grieving in a different way. She doesn’t understand this, as she says â€Å"If you had any feelings, you who dug/ With your own hand—How could you?—his little grave† (72-73). To her the only appropriate way to grieve is to show it constantly, to be living it at all times. The husband is still obviously grieving for his child, but he can’t seem to show her that he is. He still loves his child and is in mourni ng, but he doesn’t do so in the same manner as his wife does: â€Å"You—oh, you think the talk is all; I must go—/Somewhere out of this house. How can I make you—â€Å" (112-113). He doesn’t talk about it the way that she does, or

Sunday, October 27, 2019

Medical Device Regulations in the European Union

Medical Device Regulations in the European Union INTRODUCTION TO MEDICAL DEVICE REGULATIONS IN THE EUROPEAN UNION: A Medical Device under the jurisdiction of the European Union is defined as â€Å"an instrument, apparatus, appliance, material or other article, whether used alone or in combination, together with any software necessary for its proper application, which a) is intended by the manufacturer to be used for human beings for the purpose of i. diagnosis, prevention, monitoring, treatment or alleviation of disease, ii. diagnosis, monitoring, treatment, alleviation of or compensation for an injury or handicap, iii. investigation, replacement or modification of the anatomy or of a physiological process, or iv. control of conception; and b) does not achieve its principal intended action in or on the human body by pharmacological, immunological or metabolic means†.1 The clinical investigation and the subsequent introduction of a medical device in the European market is primarily regulated and governed by the MHRA (Medicines and Healthcare products Regulatory Agency) with the assistance of competent regulatory institutions called the Notified Bodies. â€Å"A Notified Body is a certification organization which the national authority (the Competent Authority) of a Member State designates to carry out one or more of the conformity assessment procedures described in the annexes of the Directives.†3 The MHRA regulates with the help of two sets of medical device regulations viz. the Statutory Instruments 2002 No.618 (Consolidated legislation) and 2003 No.1697. These legislations employ the three device directives issued by the competent authority into the european law. The directives help the manufacture in better understanding of the manufacturing and the requirments for inroduction into the market of the devices. These directives are: Directive 90/385/EEC: Active Implantable Medical Devices directive Directive 93/42/EEC: Medical Devices directive Directive 98/79/EC: In vitro Diagnostic Medical Device directive Directive 90/385/EEC: Active Implantable Medical Devices directive: This directive encompasses medical devices that are active(i.e powered) and implanted(i.e left in the human body). These include pacemakers, implantable defibrillators, implantable infusion pumps, cochlear implants and implantable neuromuscular stimulators etc. Regulations realizing the Directive came entirely into force in the United Kingdom on January 01 1995. Directive 93/42/EEC: Medical Devices directive: This directive covers an extensive array of devices from uncomplicated bandages to orthopaedic implants and high-end radiology apparatus. Regulations realizing the Directive came entirely into force in the United Kingdom on June 13 1998. Directive 98/79/EC: In vitro Diagnostic Medical Device directive: â€Å"This Directive covers any medical device, reagent, reagent product, kit, instrument, apparatus or system which is intended to be used for the invitro examination of substances derived from the human body, such as blood grouping reagents, pregnancy testing and Hepatitis B test kits. Regulations implementing the Directive came into force in the UK on 7th June 2000 with a transitional period until 7th December 2003. There is no clinical investigation system for in-vitro diagnostic medical devices. Performance evaluations of in vitro diagnostic devices that are performed outside the manufacturers premises should be notified to the UK Competent Authority in accordance with the Medical Devices Regulations 2002: Section 44.†2 The rationale backing these directives is to permit easy movement of the medical devices throughout the European Union whilst upholding high standards of device safety and up-to-the-mark quality. Classification of medical devices: Devices are classified purely based on risk associated with their use. Ranging from low risk to high risk, they are Class I, IIa, IIb and III. A classic example of a class III medical device is a cochlear implant, which is both active and implantable and thus comes under the purview of Directive 90/385/EEC: Active Implantable Medical Devices directive. Let us discuss in detail the regulatory requirments specified as per the MHRA to bring an active implantable cochlear implant into the market designated bt the European Union as the EFTA(European Free Trade Area). â€Å"Examples of AIMDs include: implantable cardiac pacemakers implantable defibrillators leads, electrodes, adaptors for 1) and 2) implantable nerve stimulators bladder stimulators sphincter stimulators diaphragm stimulators cochlear implants implantable active drug administration device catheters, sensors for 9) implantable active monitoring devices programmers, software, transmitters.†4 Cochlear Implants: â€Å"Cochlear implants are electronic hearing prostheses that bypass the damaged hearing components by providing electrical stimulation directly to the auditory nerve fibres in the cochlea. The electrical stimulation is interpreted by the brain as sound. Cochlear implants consist of an external microphone, speech processor and transmitter coil, and an internal stimulator (implanted under the skin just behind the ear) attached to a stimulation electrode which passes into the cochlea. A variation of the cochlear implant is the auditory brainstem implant where electrodes are implanted directly into the auditory area of the brainstem. This can be used in patients who do not have a functional auditory nerve.†5 The regulatory process of bringing a cochlear implant in the European market: It is mandated by law that the manufacturer who intends to bring the device into the EFTA abides by the Essential Requirments stated in the Directive 90/385/EEC: Active Implantable Medical Devices directive and demonstrate the compliance of the device with the safety and efficacy standards set forth in the directive. There are essentially two ways to do it viz. â€Å"either a compilation of the relevant scientific literature currently available on the intended purpose of the device and the techniques employed, together with, if appropriate, a written report containing a critical evaluation of the compilation; or the results and conclusions of a specifically designed clinical investigation†2 Product launch on the basis of evaluation and review of scientific literature can be considered as acceptable if equivalence can be scientifically demonstrated with a device existant in the market and routinely used in clinical practice. Equivalence has to be demonstrated w.r.t technology, critical performance, design, principles of operation, biological safety, population involved, conditions of use and clinical purpose. However, unless satisfactory evidence can be collected by means of scientific literature review, the use of a well-planned clinical trial/investigation should be considered as the best way to attest permissible levels of safety and efficacy. In case of scientific review or pre-clinical assessment, the following fees apply: Class I, IIa, or IIb other than implantable or long-term invasive:  £3,000 (Re-notification incase of objection by MHRA  £2,100). Class IIb implantable or long-term invasive, Class III, and active implantable:  £4,100 (Re-notification incase of objection by MHRA  £2,700). Applications for a proposed clinical investigation of the medical device should be made by filling the forms PCA1 and PCA2 along with the necessary information required by the clauses in the forms. Applications should be labeled clearly as â€Å"documentation only†. The use of English language is mandatory. Documentation should be clear and legible and remain so after reproduction. Electronic applications should be discussed with the MHRA. The manufacturer, for scrutiny by the MHRA should make a total of eight full submission copies available. The charges for the scrutiny of applications are laid out in the Medical Devices Regulations 2002: section 56. They are as follows: â€Å"Fees for Group A (low risk) devices are  £2,700 (initial application) or  £1,800 (resubmission). Increasing to  £3,000 and  £2,100 on 1st April 2008. Fees for Group B (high-risk) devices are  £3,800 (initial application) or  £2,400 (resubmission). Increasing to  £4,100 and  £2,700 on 1 st April 2008.†2 Applications should be forwarded to: Mrs Daniella Smolenska, Medicines Healthcare products Regulatory Agency (MHRA), European and Regulatory Affairs, Market Towers, 1 Nine Elms Lane, London, SW8 5NQ, Tel: 020 7084 3363, Email [emailprotected]. Approval from the MREC (Multi-centre Research Ethics Committee)/LREC (Local Research Ethics Committee) can be obtained along with the notification to the Competent Authority. However, a clinical investigation can begin only after approval has been obtained from the MREC/LREC and the Competent Authority has not raised an objection to the investigation within the 60 days time constraint period; or approval has been obtained from both the authoritative bodies. General Requirements: A well-defined clinical plan whose methodology and ethical considerations conforms to the standards set forth in the Medical Devices Regulations 2002: section 16 and section 29, the Active Implantable Medical Devices Directive, Annexes 6 and 7, and the Medical Devices Directive, Annexes VIII and X. Supplementary standards are set forth in Standard BS EN ISO 14155-1; 2002, â€Å"Clinical Investigation of Medical Devices for Human Subjects-part 1: General Requirements†, and BS EN ISO 14155-2:2002, â€Å"Clinical Investigation of Medical Devices for Human Subjects-part 2: Clinical Plan†. The CA should be notified incase of differences in the EU and non-EU protocols and the reasons for the same. â€Å"All applications must contain a statement (Active Implantable Medical Devices Directive: Annex 6,2.2; Medical Devices Directive: Annex VIII, 2.2): that the device in question conforms to the Essential Requirements except with regard to those aspects of the device that are to be investigated and that in respect of those aspects, every precaution has been taken to protect the health and safety of the patient. By signing this statement, the manufacturer is declaring that the device meets all of the relevant Essential Requirements, other than those subject to the investigation. Manufacturers must therefore ensure that at the time a notification is made to the Competent Authority, they have all documentation required to demonstrate conformity with the relevant Essential Requirements available for submission to the Competent Authority when requested.†2 Device information like name, model, materials used and sterilization standards etc must be provided as set forth in the directive. Pertinent information about the clinical investigation plan, investigation parameters and design, data collection and analysis methods etc. should be made available to the CA. It is strongly advised that Class III devices follow a well-designed post-marketing plan under the Medical Devices Vigilance. Extra care should be taken when labeling devices meant for clinical investigations. â€Å"All devices intended for clinical investigation must bear the wording exclusively for clinical investigation (Medical Devices Directive: annex 1, para 13.3(H) and the Active Implantable Medical Devices Directive: annex 1, 14.1).†2 Reporting of adverse incidents: â€Å"A serious adverse incident is one which: led to a death led to a serious deterioration in the health of the patient, user or others and includes: a life threatening illness or injury a permanent impairment to a body structure or function a condition requiring hospitalisation or increased length of existing hospitalisation a condition requiring otherwise unnecessary medical or surgical intervention and which might have led to death or serious deterioration in health had suitable action or intervention not taken place. This includes a malfunction of the device such that it has to be monitored more closely or temporarily or permanently taken out of service led to foetal distress, foetal death or a congenital abnormality or birth defect might have led to any of the above†2 All such incidents should be recorded and reported to the CA as set forth in the Regulation 16(10)(a) of the Medical Devices Regulations 2002 (SI 618) and Annex X of the Medical Devices Directive 93/42. Final written report: A report in conjunction with the Medical Devices Directive (Medical Devices Regulations 2002: Section 16(10) and Section 29(9)) should be submitted to the CA for devices undergoing investigation for a CE marking. However, Class III devices need to be highly regulated, before, after and during the clinical investigation. Owing to the high risks associated with their use, some say the risk can be quantified only as social and not scientific. â€Å"Risks, rather than being inherent within these implant devices, may be seen as socially derived, in processes of negotiation and conflict such as those in the case of hip and breast implants†¦.most recently, in the wake of the controversies surrounding breast implants and the 3M Capital hip, attention has been drawn to the uneven performance of notified bodies in the EU, which approve new products. This has led to the setting up of a new European Notified Bodies Operations Group (NEBOG) and calls by the MDA for all implants to be reclassified as high risk, Class III. A review of the operation of EU EMDD is also about to begin (MDA, 2001b). It thus appears that increased political scrutiny is being focused on this sector.†6 BIBLIOGRAPHY: THE MEDICAL DEVICES REGULATIONS: IMPLICATIONS ON HEALTHCARE AND OTHER RELATED ESTABLISHMENTS, BULLETIN No. 18, COMPETENT AUTHORITY (UK), Amended January 2006 EC MEDICAL DEVICES DIRECTIVES GUIDANCE FOR MANUFACTURERS ON CLINICAL INVESTIGATIONS TO BE CARRIED OUT IN THE UK, COMPETENT AUTHORITY (UK), Updated November 2007 THE NOTIFIED BODY, BULLETIN No. 6, COMPETENT AUTHORITY(UK), Amended January 2006 http://www.mhra.gov.uk/Howweregulate/Devices/ActiveImplantableMedicalDevicesDirective/index.htm, Last Date Accessed: April 08 2008 http://www.mhra.gov.uk/Safetyinformation/Generalsafetyinformationandadvice/Product-specificinformationandadvice/Cochlearimplants/index.htm, Last Date Accessed: April 08 2008 Kent, Julie and Faulkner, Alex (2002) Regulating human implant technologies in Europeunderstanding the new era in medical device regulation, Health, Risk Society, 4:2, 189 209 Medical Device Development: From Prototype to Regulatory Approval, Aaron V. Kaplan, Donald S. Baim, John J. Smith, David A. Feigal, Michael Simons, David Jefferys, Thomas J. Fogarty, Richard E. Kuntz and Martin B. Leon, 2004;109;3068-3072 Circulation, DOI: 10.1161/01.CIR.0000134695.65733.64,Circulation is published by the American Heart Association. 7272 Greenville Avenue, Dallas, TX, 72514, Copyright 2004 American Heart Association. All rights reserved. Print ISSN: 0009-7322. Online, ISSN: 1524-4539.

Friday, October 25, 2019

The Cast of Amontillado Essay -- Essays Papers

The Cast of Amontillado Edgar Allen Poe, born on January 19, 1809, is one of the greatest American writers of all time. â€Å"The story of Edgar Allen Poe’s life remains one of the most disputed and slandered in the pages of American biography, despite conscious attempts to revise the story and rehabilitate the life. Decadence and immorality, in the form of alcoholism, opium addiction, and his relationships with women, and prolific production, as a journalist, editor, poet, reviewer, critic, and fiction writer, have been emphasized as characterizing his brief life† (Lent 3). Poe’s many writings were greatly affected by his problems in life and his experiences. One such short story is â€Å"The Cask of Amontillado.† The main character, Montresor, who is vengeful, intelligent, and fakes sincerity, causes the death of Fortunato. â€Å"The Cask of Amontillado† starts out with Montresor, the narrator, saying, â€Å"The thousand injuries of Fortunato I had borne as I best could; but when he ventured upon insult, I vowed revenge.† Simply by reading the first sentence of the story, it is easy to see that Montresor is vengeful and plans to get â€Å"revenge† on Fortunato. Montresor also has a coat of arms which is, â€Å"A huge human foot d’or, in a field azure; the foot crushes a serpent rampant whose fangs are imbedded in the heel,† with a motto of, â€Å"Nemo me impune lacessit,† which stands for no one attacks me with impunity. The coat of arms and the family motto both suggest retribution. The ar...

Thursday, October 24, 2019

Spontaneous Recovery: Reflections of a Smoker

Definition: Spontaneous Recovery is the tendency of a learned behavior to recover from the extinction after a rest period. (Schacter, Gilbert, Wegner 2009). In order to better understand this definition I had to do more reading on the basic principles of learning and behaviorism. The definition for learning is some experience that results in a relatively permanent change in the state in the learner. (Schacter, Gilbert, Wegner 2009) In order for one to have a spontaneous recovery you must first have learned the habit, such as smoking. Next is the habitation, which is a general process in which repeated or prolonged exposure to a stimulus results in a gradual reduction in responding (Schacter, Gilbert, Wegner 2009) This would be the learned smokers getting use to the taste of nicotine, the smell of the smoke and ultimately the mood or mental state associated with the effects of the cigarette. I have been a smoker since I was about fourteen years old. At first I began smoking due to peer pressure. I was not truly addicted till the year 2003 when everyone I worked with would gather and smoke after out shift to relax and reflect on our night. In 2008 I quit smoking with the use of the medication called Chantix with complete success. Unfortunately, I picked it up again when I had a roommate move who smoked. Now even though I hadn’t smoked in over a year the craving came back, which was the spontaneous recovery. I hadn’t smoked In over a year and found myself under stress and quickly went back to the habit (habituation) of smoking again and have been doing so for the last year. Now, I find myself with another prescription for Chantix but I know that unless I control my urges after I quit. I will forever have the learned behaviorism of a smoker and must always remember that spontaneous recovery will be right around the corner if I allow myself to fall back into the habit of smoking.

Wednesday, October 23, 2019

Managing Profitable Customer Relationship

Chapter 1 Marketing: Managing Profitable Customer Relationships GENERAL CONTENT: Multiple-Choice Questions 1. Central to any definition of marketing is _____. a. demand management b. transactions c. customer relationships d. making a sale e. making a profit (Answer: c; p. 5; Easy) 2. All of the following are accurate descriptions of modern marketing today, except which one? a. Marketing is creation of value for customers. b. Marketing is customer satisfaction at a profit. c. Selling and advertising are synonymous with marketing. d. Marketing involves building and managing profitable customer relationships. e. None of the above statements is true. (Answer: c; p. 5; Easy)3. Like NASCAR, successful companies recognize a crucial dimension of an outstanding marketing company to be _____. a. a strong customer focus b. a relentless pursuit of customer needs c. customer relationships built by everyone in the organization d. all of the above e. none of the above (Answer: d; p. 5; Moderate)4. _____ is defined as a social and managerial process by which individuals and organizations obtain what they need and want through value creation. a. Selling b. Advertising c. Barter d. Marketing e. None of the above is correct. Answer: d; p. 5; Challenging)5. Society and culture sh ape the basic form of human needs called _____. a. needs b. wants c. demands d. value e. an exchange (Answer: b; p. 6; Moderate)6. When backed by buying power, wants become _____. a. social needs b. demands c. physical needs d. self-esteem needs e. exchanges (Answer: b; p. 6; Easy)7. The fundamental reason America’s most admired firms, including Southwest Airlines and Harley-Davidson, conduct extensive research is to _____. a. maximize profits b. increase market share c. increase sales d. understand customers’ needs, wants, and demands e. undermine competitors Answer: d; p. 6; Moderate)8. _____ refers to sellers being preoccupied with their own products and losing sight of underlying consumer needs. a. Selling myopia b. Marketing c. Selling d. Marketing myopia e. Share of customer (Answer: d; p. 7; Moderate)9. American brand icons, including Harley-Davidson, Coca-Cola, and Nike, create deep brand meanings for consumers and do not suffer from _____. a. short-term losses b. long-term losses c. competitive threats d. marketing myopia e. planning problems (Answer: d; p. 7; Moderate)10. _____ are key building blocks for developing and managing customer relationships. a.Consumer expectations and customer satisfaction b. Customer choices and product offers c. Product performance and customer value d. Customer value and customer satisfaction e. Strategic plans (Answer: d; p. 8; Challenging)11. NASCAR’s primary obsession is to deliver a special _____ to every customer. a. assortment of products b. book store c. experience d. set of recommendations e. car (Answer: c; p. 7; Moderate)12. All of the following phrases reflect the marketing concept, except which one? a. We don’t have a Marketing Department, we have a Customer Department. b. We make it happen for you. c. We stay close to customers. d. Putting profits ahead of customer needs is critical to the health of the firm. e. Customers are important. (Answer: d; p. 11; Easy)13. An example of a(n) _____ is when a customer pays Circuit City $250 and receives a television set in return. a. exchange b. transaction c. market d. segment e. scam (Answer: b; p. 9; Easy)14. _____ is the set of actual and potential buyers of a product. a. A market b. An audience c. A group d. A segment e. An exchange (Answer: a; p. 8; Moderate)15. Which of the following phrases reflects the marketing concept? a. The supplier is king. b. Marketing should be viewed as hunting and not gardening. . This is what I make, won’t you please buy it? d. This is what I want, won’t you please make it? e. None of the above. (Answer: d; p. 11; Challenging)16. The art and science of choosing target markets and building profitable relationships with them is called _____. a. marketing management b. positioning c. segmentation d. selling e. managing the marketing effort (Answer: a; p. 9; Moderate)17. _____ is(are) the set of benefits a company promises to deliver its consumers to satisfy their needs. a. A money-back guarantee b. Low prices c. Good customer service d. A value proposition e. An attribute (Answer: d; p. 9; Moderate)18. All of the following phrases reflect a firm’s value proposition, except which one? a. Altoids is positioned as â€Å"the curiously strong mint. † b. Porsche promises driving performance and excitement. c. Cheer laundry detergent promises powerful cleaning at all temperatures. d. All of the above are correct. e. None of the above is correct. (Answer: d; p. 9; Moderate)19. The _____ is a useful philosophy in situations when the product’s cost is too high and marketers look for ways to bring it down. a. selling concept b. product concept c. production concept d. marketing concept e. A and D (Answer: c; p. 10; Challenging)20. Henry Ford’s philosophy was to perfect the Model-T so that its cost could be reduced further for increased consumer affordability. This reflects the _____. a. product concept b. marketing concept c. marke ting mix d. production concept e. selling concept (Answer: d; p. 10; Moderate)21. To avoid traffic gridlock in large metro areas, _____ is undertaken to encourage commuters to carpool and use mass transit. a. target marketing b. market segmentation c. demarketing d. marketing e. the production concept (Answer: c; p. 9; Challenging)22. Selecting which segments to serve is called _____. a. market segmentation b. positioning c. customization . target marketing e. managing the marketing effort (Answer: d; p. 9; Moderate)23. When Wal-Mart profitably targets buyers who value savings, it is an example of _____. a. convenience b. value pricing c. market segmentation d. target marketing e. value packing (Answer: d; p. 9; Easy)24. All of the following phrases reflect the definition of target marketing, except which one? a. Disney targets persons in all stages of the life cycle. b. Porsche profitably targets affluent professionals. c. Dollar Stores profitably target families with modest means. d. The Book of the Month Club customizes offers based on a member’s previous selections. . Charlie Cheese Pizza Factory targets children. (Answer: a; p. 9; Moderate)25. â€Å"Build a better mousetrap and the world will beat a path to your door† reflects the _____. a. production concept b. marketing concept c. selling concept d. product concept e. target marketing (Answer: d; p. 10; Challenging)26. Which operating philosophy is practiced by the American Red Cross to solicit blood donations? a. The marketing concept. b. The product concept. c. The production concept. d. The selling concept. e. None of the above. (Answer: d; p. 10; Challenging)27. Firms follow the _____ when they face overcapacity. a. product concept b. elling concept c. production concept d. marketing concept e. A and C (Answer: b; p. 10; Challenging)28. Railroads were once operated based on the thinking that users wanted trains rather than transportation, overlooking the challenge of other modes of tra nsportation. This reflects the _____. a. product concept b. production concept c. selling concept d. marketing concept e. none of the above (Answer: a; p. 10; Moderate)29. According to the authors of your text, _____ is viewed not as â€Å"hunting,† but as â€Å"gardening. † That is, a firm has to find the right products for its customers. a. selling b. production c. marketing d. retailing e. dvertising (Answer: c; p. 11; Moderate)30. The _____ starts with the factory, focusing on the company’s existing products; it calls for heavy selling and promotion to obtain profitable sales. a. marketing concept b. production concept c. product concept d. selling concept e. company advertising (Answer: d; p. 10; Moderate)31. According to the authors of your text, fast-food restaurants offer tasty and convenient food at affordable prices; they contribute to a national obesity epidemic that harms consumer health and causes environmental problems in the long run. This stateme nt reflects that they often overlook the _____ business philosophy. . marketing concept b. product concept c. production concept d. societal marketing concept e. new-idea (Answer: d; p. 11; Moderate)32. The set of marketing tools a firm uses to implement its marketing strategy is called the _____. a. promotion mix b. product mix c. marketing mix d. TQM e. marketing effort (Answer: c; p. 13; Moderate)33. _____ is defined as the customer’s evaluation of the difference between all the benefits and all the costs of a marketing offer relative to those of competing offers. a. Customer relationship management b. Customer satisfaction c. TQM d. Customer perceived value e. Marketing myopia (Answer: d; p. 4; Easy)34. Building, keeping, and growing profitable value-laden relationships with all customers of a company is called _____. a. customer lifetime value b. customer perceived value c. customer relationship management d. database marketing e. societal marketing (Answer: c; p. 14; Ea sy)35. Delivering superior customer value and customer satisfaction are the two keys to building lasting _____. a. customer satisfaction b. customer databases c. market share d. customer relationships e. profits (Answer: d; p. 14; Moderate)36. All of the statements below reflect the definition of customer-perceived value, except which one? . Alex brought home his Lexus for $45,000; he luxuriated in the handling and smell of the leather interior. b. FedEx offers reliable package delivery at a reasonable price. c. The benefits of undergraduate tuition at state schools are judged to be reasonable and fair in comparison to competing private schools. d. The benefits of diet soft drinks are judged to be reasonable and fair in comparison to other types of soft drinks. e. All of the above are correct. (Answer: e; p. 14; Moderate)37. FedEx offers its customers fast and reliable package delivery. When FedEx customers weigh these aforementioned benefits against the monetary and psychic costs o f using the service, they are acting upon _____. a. loyalty b. relationship marketing c. perceived customer value d. social relationships e. a societal marketing campaign (Answer: c; p. 14; Challenging)38. Airlines offer frequent flier marketing programs to build value and satisfaction into the customer relationship. Using this approach, airlines add _____ to the customer relationship. a. social benefits b. structural benefits c. financial benefits d. excitement e. add-ons (Answer: c; p. 15; Challenging)39.Many banks are leading the way in using customer profitability analysis to weed out losing customers and target winning ones for pampering. This is called _____. a. customer relationship management b. positioning c. database marketing d. selective relationship management e. prospecting (Answer: d; p. 16; Challenging)40. Through _____, many companies today are strengthening their connections to partners all along the channel, from raw materials to components to final products that are carried to final buyers. a. supply chain management b. direct marketing c. partnership relationship marketing d. customized marketing e. deviated marketing Answer: a; p. 19; Easy)41. Pooling resources with other firms in order to succeed beyond managing the supply chain illustrates the _____ partnership. a. management contracting b. licensing c. supply chain management d. strategic alliance e. exporting (Answer: d; p. 19; Challenging)42. Stew Leonard, owner/operator of supermarkets, reacts adversely to losing a single customer sale. He feels that this amounts to losing the entire stream of future purchases that a customer is likely to make if he/she remains in the area. This is an illustration of _____. a. share of customer b. market share c. profitability d. customer lifetime value . market share maintenance (Answer: d; p. 20; Challenging)43. Amazon. com leverages relationships with its 35 million customers by offering them music, videos, gifts, toys, consumer electronics, and office products, among other product items. Based on previous purchase history, the company recommends related CDs, books, or videos that might be of interest. This helps Amazon. com capture a greater _____. a. market share b. customer lifetime value c. share of customer d. profitability e. customer base (Answer: c; p. 21; Challenging)44. _____ is the total combined customer lifetime values of all the company’s customers. . Share of customer b. Customer lifetime value c. Customer equity d. Profitability e. Share of market (Answer: c; p. 21; Challenging)45. Current sales and market share reflect a firm’s past performance while _____ reflects the future. a. customer lifetime value b. share of customer c. profitability d. customer equity e. growth rate (Answer: d; p. 21; Moderate)46. Banks classify customers into one of four relationship groups, according to their potential profitability and projected loyalty. JPMorgan Chase Bank wishes to examine its database and identif y customers who are profitable but not loyal. According to the authors of the text, it plans to use promotional blitzes to attract the group called _____. a. barnacles b. true friends c. strangers d. butterflies e. fools (Answer: d; p. 23; Challenging)47. The ultimate aim of customer relationship management is to produce _____. a. customer equity b. market share c. sales volume d. a reliable database e. profits (Answer: a; p. 21; Moderate)48. Which of the following statements about how the Internet is impacting lives everywhere is most accurate? a. Companies are cautiously using the Internet to build closer relationships with customers and marketing partners alike. . The Internet is still in its infancy with few consumers buying products/services online. c. The Internet allows anytime, anywhere connections to information, entertainment, and communication. d. If consumer e-commerce looks promising, business-to-business e-commerce is just plain declining. e. A and B (Answer: c; p. 26; Easy)49. The rapid pace of _____ has allowed companies to greatly expand their geographical market coverage, purchasing, and manufacturing. a. technology b. change c. travel d. globalization e. none of the above (Answer: d; p. 26; Moderate)50. Perhaps the most dramatic new technology today is _____. a. Microsoft Windows XP b. AOL c. the Internet d. all of the above e. none of the above (Answer: c; p. 26; Easy)51. Ben & Jerry’s challenges all stakeholders, including employees, top management, and even ice cream scoopers in their stores, to include concern for individual and community welfare in their day-to-day decisions. Actions by companies to do well by doing good reflects _____. a. ethics b. social responsibility c. profit marketing d. marketing e. myopia (Answer: b; p. 28; Moderate)52. When a church targets different demographic groups to increase attendance, it is an example of _____. a. for-profit marketing b. ot-for-profit marketing c. mindless marketing d. ethics in marketing e. societal marketing (Answer: b; p. 28; Moderate)53. The goal of customer relationship management is to produce _____. a. revenues b. profits c. customer equity d. a database of customers e. all of the above (Answer: c; p. 21; Moderate)54. To create customer value and build strong customer relationships, marketers know they cannot go it alone. Hence, in order to succeed in the long run, they practice _____. a. partner relationship management b. database marketing c. designing attractive websites d. customer equity e. all of the above (Answer: a; p. 19; Challenging)55. The success of a firm hinges upon the performance of the entire _____. a. marketing department’s effort b. supply chain c. product mix offerings d. organizational structure e. industry (Answer: b; p. 19; Moderate)56. The authors of your text classify customers into one of four relationship groups, according to their profitability and projected loyalty. _____ characterize the group with the highest pro fit potential and strong loyalty. a. Barnacles b. Strangers c. Butterflies d. True believers e. Best friends (Answer: d; p. 23; Easy)57. _____ is the act of obtaining a desired object from someone by offering something in return. . A transaction b. Exchanging c. Bribing d. Valuing e. Donating (Answer: b; p. 8; Easy)58. In the case of excess demand, _____ may be required to reduce the number of customers or to shift demand temporarily or permanently. a. marketing b. demarketing c. value marketing d. surplusing e. all of the above (Answer: b; p. 9; Easy)59. The _____ concept holds that achieving organizational goals depends on knowing the needs and wants of target markets and delivering the desired satisfaction better than competitors do. a. product b. production c. selling d. equity e. marketing (Answer: e; p. 11; Easy)60.The _____ concept holds that firms must strive to deliver value to customers in a way that maintains or improves both the consumer’s and society’s wel l being. a. marketing b. selling c. product d. societal marketing e. equity (Answer: d; p. 11; Easy)61. The _____ of a Taco Bell customer exceeds $12,000. a. market share b. CRM c. customer lifetime value d. share of stomach e. salary (Answer: c; p. 21; Moderate)62. For which of the following reasons are marketers everywhere using the Internet? a. To build closer relationships. b. To access new marketspaces. c. To attract new customers. d. To show off their technological skills. . A, B, and C (Answer: e; p. 26; Easy)63. You have learned at work that today’s successful companies at all levels have one thing in common: they are strongly customer focused and heavily committed to _____. a. obtaining the best CEOs b. increasing wealth to stockholders c. marketing d. employee motivation e. earning profits (Answer: c; p. 5; Moderate)64. The twofold goal of marketing is to attract new customers by promising superior value and to _____. a. keep and grow current customers by delivering satisfaction b. keep and grow current customers by delivering competitive pricing c. eep and grow current customers by delivering friendly service d. keep and grow current customers by delivering vast product assortment e. all of the above (Answer: a; p. 5; Moderate)65. You have learned from experience as well as from this course that the most basic concept underlying marketing is that of _____. a. selling and advertising b. customer satisfaction c. retaining customers d. human needs e. fulfilling consumer wants (Answer: d; p. 6; Easy)66. As a new assistant marketing manager trainee, you learn in an orientation meeting that _____ are the form human needs take as they are shaped by culture and individual personality. . wants b. demands c. self-concepts d. desires e. icons (Answer: a; p. 6; Easy)67. What do companies call a set of benefits that they promise to consumers to satisfy their needs? a. marketing offer b. value proposition c. demand satisfaction d. need proposition e. evoke d set (Answer: b; p. 9; Moderate)68. Most firms practice the selling concept when they face _____. a. a crisis b. a recession c. fierce competition d. overcapacity e. marketing myopia (Answer: d; p. 10; Moderate)69. Herb Kelleher of Southwest Airlines used the marketing concept in his successful organization.His perspective of having a customer department uses a(n) _____ perspective. a. outside-in b. external c. inside-out d. modern e. traditional (Answer: a; p. 11; Challenging)70. Customer-driven marketing usually works well when _____ and when customers _____. a. a clear need exists; are easy to identify b. customers know what they want; can afford it c. a firm can deliver the goods desired; are thoroughly researched d. a clear need exists; know what they want e. a want exists; cannot afford it (Answer: d; p. 11; Challenging)71. The societal marketing concept seeks to establish a balance between consumer short-run wants and consumer _____. . short-run costs and profits b. short-ru n ethics c. long-run welfare d. health e. value propositions (Answer: c; p. 12; Moderate)72. _____ is an important concept when we realize that losing a customer means losing more than a single sale. It means losing the entire stream of purchases that the customer would make over a lifetime of patronage. a. Heuristics b. Net profit c. Customer lifetime value d. Relationship marketing e. Market share (Answer: c; p. 20; Moderate)73. Customers buy from stores and firms that offer the highest _____. a. value for the dollar b. customer perceived value c. level of customer satisfaction . company image e. A and C (Answer: b; p. 14; Challenging)74. Is the following statement true? Clearly, the more loyal the firm’s customers, the higher the firm’s customer equity. a. No. b. Yes. c. Maybe. d. Cannot tell accurately. e. Only if the value proposition is understood. (Answer: b; p. 21; Easy)75. Many not-for-profit organizations are facing huge operating deficits that they must cove r by more aggressive _____. a. volunteer service b. customer service c. advertising d. donor marketing e. social marketing campaigns (Answer: d; p. 29; Moderate) True/False76. Selling is managing profitable customer relationships. Answer: False; p. 5; Moderate)77. Product, price, place, and promotion make up the elements of a firm’s marketing mix. (Answer: True; p. 13; Easy)78. The twofold goal of marketing is to attract new customers by promising superior value and to keep and grow current customers by delivering satisfaction. (Answer: True; p. 5; Challenging)79. Human needs are shaped by culture and individual personality. (Answer: False; p. 6; Moderate)80. When backed by buying power, wants become demands. (Answer: True; p. 6; Moderate)81. When backed by buying power, needs become demands. (Answer: False; p. 6; Moderate)82. Marketing offers are limited to physical products. (Answer: False; p. 7; Moderate)83. Marketing offers include products, services, information, or expe riences offered to a market to satisfy a need or want. (Answer: False; p. 7; Moderate)84. When sellers focus on existing needs and lose sight of underlying customer wants, they suffer from marketing myopia. (Answer: False; p. 7; Challenging)85. An exchange is the core concept of marketing, whereas a transaction is marketing’s unit of measurement. (Answer: True; p. 8; Moderate)86. Marketers of products, services, and ideas only practice marketing, whereas buyers do not. Answer: False; p. 8; Moderate)87. Who is our target market and what’s our value proposition are two important questions underlying marketing strategy. (Answer: True; p. 9; Moderate)88. Market segmentation is the process of seeking fewer customers and reduced demand for profit maximization only. (Answer: False; p. 9; Challenging)89. Demarketing is a marketing philosophy focused upon product differentiation and positioning. (Answer: False; p. 9; Moderate)90. The production concept and product concept are t wo philosophies that can both lead to marketing myopia. (Answer: True; p. 10; Challenging)91. When railroad companies thought that users wanted trains rather than transportation and overlooked the growing challenge of other modes of transportation they were following the selling concept. (Answer: False; p. 10; Moderate)92. Most firms follow the production concept when they face overcapacity. (Answer: False; p. 10; Challenging)93. The societal marketing concept calls on marketers to balance consumer wants and desires, company profits, and society’s interest. (Answer: True; p. 12; Moderate)94. Customer Relationship Management (CRM) is nothing more than a customer data management activity. (Answer: False; p. 14; Moderate)95. Delivering superior customer value and customer satisfaction are the two keys to building lasting customer relationships. (Answer: True; p. 14; Moderate)96. Customer value is defined as the customer’s evaluation of the perceived difference between all the benefits and all the costs of a marketing offer relative to those of competing offers. (Answer: True; p. 14; Moderate)97. Customer-perceived value depends on the product’s perceived performance relative to a buyer’s expectations. (Answer: False; p. 14; Moderate)98. The simplest definition of modern marketing is managing profitable customer relationships. (Answer: True; p. 5; Easy)99.The difference between human needs and wants is that needs are states of felt deprivation. (Answer: True; p. 6; Easy)100. Smart marketers look beyond the attributes of the products and services they sell. They create brand experiences for consumers. (Answer: True; p. 7; Moderate)101. Marketing management is interested in serving all customers in every way to remain competitive in today’s markets. (Answer: False; p. 9; Moderate)102. At times it becomes necessary to reduce demand for some products and services. When the government tries to reduce smoking of tobacco products, it ad ds more tax to the products and is practicing demarketing. Answer: True; p. 9; Easy)103. Amy’s law office has developed a new format and wording for wills. The staff believes they offer the most in quality, performance, and innovative features. Her law office is practicing the production concept. (Answer: False; p. 10; Moderate)104. The selling concept holds that consumers will not buy enough of the firm’s products unless it undertakes a large-scale selling and promotion effort. (Answer: True; p. 10; Easy)105. The major difference between customer-driving marketing and customer-driven marketing is that the former considers only existing needs. Answer: False; p. 11; Easy) Essay106. Explain the five marketplace concepts. The core marketplace concepts are: needs, wants, and demands; marketing offers; value and satisfaction; exchanges, transactions, and relationships; and markets. Addressing customer needs and want is at the very heart of the marketing concept. The four el ements of the marketing mix help firms meet the challenges of value creation, customer satisfaction, and to establish meaningful and profitable relationships. (p. 6; Moderate)107. Explain how marketers create brand experiences beyond selling products/services. Strategic thinking underlies creating meaningful and purposeful experiences and relationships for customers. In creating brand experiences, marketers have successfully demonstrated that to differentiate their offer from their competitors, they have to connect with their customers at various levels. (p. 7; Moderate)108. Compare the selling and marketing concepts under which organizations carry out their marketing strategies. List the key components of each philosophy. The selling concept reflects an inside-out philosophy and the marketing concept takes an outside-in perspective. The selling concept is practiced when firms face overcapacity. When consumers do not buy enough products on their own, companies coerce them into buyin g more by undertaking a large-scale selling and promotion effort. The marketing concept, on the other hand, is a three-pronged philosophy based upon: satisfaction of customer needs and wants, integration of resources both within and outside the firm,, and profit maximization. (p. 10; Easy)109. What is the societal marketing concept? Explain. According to this concept, firms today and in the future will survive if they take underlying consumer needs and society’s well being into account over the long term. In setting their marketing strategies, marketers today need to balance company profits, consumer wants, and society’s interests. From day one, when marketing decisions are made, firms need to put people and society before profits. (p. 11; Easy)110. One of the major developments in marketing can be summed up on one buzzword: relationships. Define customer relationship management and its associated strategies for building long-term relationships. Customer Relationship M anagement (CRM) is the process of building and maintaining profitable customer relationships by delivering superior customer value and satisfaction.Companies develop customer relationships with target markets at multiple levels. The most basic form of a relationship for mass-marketed products/services is through a Web site, sales promotion offer, or a 1-800 customer-response number. At the other end of the spectrum, companies like Amazon. com create full partnerships with key customers. Other marketers work closely with retailers, for example. Some marketers use tools such as financial benefits like rewards based on frequency of purchase. Other tools include social benefits, like offering key customers the opportunity to network and create communities.Another approach adds structural ties to the aforementioned financial and social benefits. Hence, to retain current customers and remain profitable, companies today are going beyond transactional marketing to customer relationship mana gement. The key is to create and sustain relationships for the long term. (p. 14; Moderate)111. The aim of customer relationship management is to create not just customer satisfaction, but customer delight. Explain. Customer satisfaction cannot be taken for granted. Because brand loyalty is dependent upon strong customer satisfaction, companies strive to retain current customers.Losing a customer once might mean losing the customer for life. Firms like Taco Bell and Home Depot, for example, look at a stream of purchases a customer is likely to make over his/her lifetime. Therefore each sale is critical to the long-term success of a relationship. Because the ultimate aim of customer lifetime value is to create share of customer, firms today create customer delight by over delivering and creating emotional relationships with key customers. (p. 15; Moderate)112. Define customer equity. Customer equity is the sum of the lifetime values of all the company’s customers. Customer equ ity is dependent upon customer loyalty by a firm’s profitable customers. Because customer equity is a reflection of a company’s future, companies must manage it carefully. (p. 21; Easy)113. Explain how the Internet has transformed the way in which we do business today. The Internet links individuals and businesses of all types to each other. â€Å"Bricks and Mortar† companies of the past are now â€Å"clicks and mortar† companies today. Manufacturing firms today are linked to their suppliers and customers to build closer relationships. The Internet allows firms access to exciting new marketspaces. The Internet has spawned an entirely new breed of â€Å"click only† companies—the so-called â€Å"dot-coms. † The post-Internet frenzy of the late 1990s has introduced companies that are both savvy and face promising futures. (p. 26; Easy)114. Describe the impact of globalization on marketing today. Marketers everywhere have been surrounded by global competition for over two decades now. Regional trade agreements, such as NAFTA, have transformed competition and economic cooperation today. Geographical and cultural distances, in addition, have shrunk with the advent of technology, the Internet and new product introduction. Domestic firms in countries such as India have had to contend and compete with U. S. multinational firms for market share, revenues, and profits. Firms worldwide are sourcing their products from different corners of the globe. (p. 26; Moderate)115. Analyze the major challenges facing marketers heading into the new â€Å"connected† millennium. Marketers must connect faster and better with customers. The latest technologies must be used to ensure delivery of time-based competition. Web sites and e-commerce must be fine-tuned to connect with more carefully selected customers.Many companies are connecting directly with customers to customize their mix of products and services. Partnership relations hip and supply chain management must be built with strategic alliances to make those domestic and global challenges. (p. 28; Moderate)APPLICATION CONTENT: Multiple-Choice Questions116. Shawn McCork has an interesting job. He is involved in getting, keeping, and growing customers through creating, delivering, and communicating superior customer value. What is Shawn’s job? a. General manager. b. Supervisor. c. Marketing manager. d. Sales manager. e. Top manager. (Answer: c; p. ; Challenging)117. The marketing manager at Sunshine Car Washes is holding a training session for new employees. She stresses that perhaps the most important concept of modern marketing is _____. a. customer relationship management b. e-mail advertising c. a quality Web site d. properly trained sales people e. low prices (Answer: a; p. 9; Moderate)118. Greg Williams now has the buying power to purchase the computer system he has wanted for the last six months. Greg’s want now has become a _____. a. need b. necessity c. demand d. satisfier e. none of the above (Answer: c; p. 7; Easy)119. After surveying all 3,500 customers by e-mail, Best Value Stores learned that its customers favor high quality, performance, and innovative features. Best Value’s customers were surveyed about _____. a. product concept b. production concept c. customer satisfaction d. marketing concept e. promotion concept (Answer: a; p. 10; Challenging)120. Jolene’s firm believes that consumers will not buy enough of the firm’s products unless the firm undertakes a large-scale selling and promotion effort. Jolene’s firm is practicing the _____. a. production concept b. marketing concept c. selling concept d. relationship concept . social advertising campaign (Answer: c; p. 10; Easy)121. Jonathan Nash works in sales for a telemarketing firm. His firm uses the selling concept, which take a(n) _____ approach. a. outside-in b. myopic c. inside-out d. marketing concept e. customer servi ce (Answer: c; p. 11; Easy)122. Marie Ortiz enjoys her work at Futuristic Designs, Inc. Her organization understands customer needs even better than customers themselves do and creates products and services that will meet existing and latent needs, now and in the future. Marie’s firm practices _____ marketing. a. customer-driven b. customer-driving c. elationship d. donor e. none of the above (Answer: b; p. 11; Challenging)123. You find yourself in a new job. Your marketing manager is heavily involved in the process of building and maintaining profitable customer relationships by delivering superior customer value and satisfaction. Your manager is concerned with which one of the following? a. Database management. b. Web site hits. c. Relationship management. d. Donor marketing. e. Customer relationship management. (Answer: e; p. 14; Moderate)124. Tommy Gray attempts to deliver customer satisfaction every day in his Audio Expressions installation business. He is a smart operat or who knows that the key to this goal is to match _____ with _____. a. customer expectations; competitive prices b. company performance; competition c. customer expectations; company performance d. company performance; unique products e. relationship building; promotional tools (Answer: c; p. 14; Moderate)125. You have just read a report that alarms you. According to the American Customer Satisfaction Index, which of the following conditions exists relative to overall customer satisfaction with U. S. manufacturing and service industries? a. It has increased slightly. b. It has increased very much. c. It has remained steady. d. It has decreased slightly. e. It has decreased dramatically. (Answer: d; p. 20; Easy)126. Shania works hard with her Internet customers to create an emotional relationship for her customers with the products and services that she and her staff sell. She and her staff have created _____ by going beyond the expected. a. customer delight b. customer satisfaction c. customer equity d. customer value e. customer loyalty (Answer: a; p. 14; Challenging)127. Karrie Romanov wants to capture the full essence of customer relationship management. Which of the following will Karrie take into consideration? a. Own the customers for life. b. Capture their lifetime value. c. Building overall customer equity. d. All of the above. e. None of the above. (Answer: d; p. 14; Moderate)128. Some firms find themselves in markets with many low-margin customers. As assistant marketing director, what type of relationship would you develop with these customers? a. Full partnerships. b. Basic relationships. c. Relationship marketing. d. Key customer marketing. e. Lifetime value. (Answer: b; p. 15; Moderate)129. You have just read a report in a leading business magazine. It stated that the major marketing developments as we enter the new millennium can be summed up in what single theme? a. Relationship marketing. b. Connecting. c. Partnering. d. Networking. e. Custom er equity. (Answer: b; p. 23; Challenging)130. You have just been told by your supervisor at work that a new economy has emerged. What is the technology behind this new force? a. The Internet. b. Web sites. c. Voice mail. d. Cell phones. e. Simultaneous engineering. (Answer: a; p. 26; Easy)131. Pete Sanchez has just realized something that he needs to tell his marketing manager at work. Pete knows that today few firms still practice rue _____. a. production orientation b. sales orientation c. mass marketing d. quality orientation e. marketing segmenting (Answer: c; p. 16; Moderate)132. Suzie Chan strengthens her company’s connections with partners all along the supply chain. What type of management is she using? a. Outside partnering. b. Supplier connecting. c. Mentoring. d. Supply chain. e. Channeling. (Answer: d; p. 19; Easy)133. ABC Corporation realizes that they need partners to go beyond supply chain management. What do we call this association? a. Strategic alliances. b . strategic planning. c. Partnering. d. Mutual reciprocity. e. Reengineering. Answer: a; p. 19; Moderate)134. Sally purchased Brand X lotion. In analyzing the product’s perceived performance against her expectations, Sally was measuring her level of _____. a. customer perceived value b. customer satisfaction c. exchange d. demand e. customer lifetime value (Answer: b; p. 14; Moderate)135. Members of the sales team at Dekko International visit only prospective customers who purchase a minimum of $50,000 of insulated wire per year. Dekko is using _____. a. selective relationship management b. a frequency marketing program c. a club marketing program d. demarketing e. a value proposition (Answer: a; p. 6; Easy) Short Answer136. What is the twofold goal of marketing? The twofold goal of marketing is to attract new customers by promising superior value and to keep and grow current customers by delivering satisfaction. (p. 5; Easy)137. Culture and individual personality shape human needs into wants. What transforms wants into demands? Wants become demands when backed by purchasing power. (p. 6; Moderate)138. How might a seller avoid marketing myopia? Sellers should consider the particular benefits and experiences produced by their products, not just pay attention to the specific products they offer. (p. 7; Moderate)139. How might a manufacturer of tents and camping equipment, for example, create brand experiences for consumers? Such manufacturers might produce tents, sleeping bags, cooking equipment, and so forth that allow consumers to benefit from the numerous products available to campers. (p. 7; Easy)140. The main elements of a modern marketing system relies on profitable relationships all along the way. Considering this, what might Wal-Mart rely on in order to offer low prices? Wal-Mart must rely on suppliers that will provide merchandise at low costs. (p. 8; Moderate)141. How might a marketer define its value proposition? In considering its value propos ition, a marketer will look at how the firm can best serve the customers and how it can differentiate itself in the marketplace. (p. 9; Easy)142. When demand for Beanie Babies was at its highest, manufacturers purposefully maintained strong demand by limiting supply that drove price up. Explain how such manufacturers were not carrying out the production concept. The production concept holds that consumers favor products that are available and affordable. With this concept, manufacturers work to increase production and improve manufacturing efficiency. (p. 10; Challenging)143. Company X carries a vast surplus of office supplies; thus, the company follows the selling concept. Explain how customer relationships may be lost in the process. The company’s aim is to sell the supplies rather than make what the market wants; such a strategy creates sales transactions but not long-term relationships. (p. 10; Challenging)144. Many companies, such as Southwest Airlines, take an outside-i n perspective. How do such companies address their customers’ desires? Outside-in companies are customer driven; they find the right products for their customers rather than the right customers for their products. (p. 11; Moderate)145. A nineteenth-century street vendor in London sang, â€Å"Who will buy my fresh, red roses? † Did the vendor take an outside-in or inside-out perspective? Explain. The vendor’s approach was inside-out. The roses were picked and available. The vendor’s job was then to attract willing buyers. (p. 11; Challenging)146. When a vendor has product available and needs to find customers who are willing to buy, is a production concept, product concept, or selling concept being practiced? Explain. A selling concept is used when the vendor has available product and needs to find customers who are willing to buy. (p. 10; Moderate)147. Explain why electronics and pharmaceuticals manufacturers may use customer-driving marketing. In such in dustries, consumers do not know exactly what new products are available; therefore, consumers rely on such firms to tell them what they need. (p. 11; Moderate)148. Company ABC implements its marketing strategy through a well-defined marketing mix. What elements are being addressed in the marketing mix? Company ABC has created a marketing offer (product), determined a selling price, decided how to distribute (place) the offer, and communicated with the target customer about the offer (promotion). (p. 13; Easy)149. Explain how storing customer information in a database might better prepare Saturn in customer relationship management (CRM). Managing detailed information about customers may allow Saturn to design new models around customer demographics and desires for specific features. These â€Å"touchpoints† can be the key to long-term customer loyalty. (p. 14; Moderate)150. What determines whether sellers create basic relationships or full partnerships with customers? A compan y with many low-margin customers develops basic relationships; a company with just a few high-margin customers relies on full partnerships. (p. 15; Challenging)151. Explain how a supermarket owner might consider customer lifetime value when a disgruntled customer leaves the store dissatisfied. The owner may view the situation as a $50,000 loss if, for example, each customer spends about $100 per week, shops 50 weeks per year, and remains in the area for about 10 years. Customer lifetime value includes the long-term value of the customer. (p. 20; Easy)152. How can a marketer increase â€Å"share of customer†? The marketer can offer greater variety to customers; in addition, the marketer can train employees to cross-sell and up-sell in order to market more products and services to existing customers. p. 21; Easy)153. In classifying customers into relationship groups, explain what marketers can expect from â€Å"butterflies. † â€Å"Butterflies† are profitable but not loyal. Marketers should enjoy â€Å"butterflies† â€Å"for the moment† because they soon flutter off. Marketers should create profitable and satisfying transactions with â€Å"butterflies,† then cease investing in them until the next time around. (p. 23; Moderate)154. If a firm practices â€Å"caring capitalism† in its social responsibility efforts, as does Ben & Jerry’s and Saturn, where does the firm place its focus? Such firms distinguish themselves by being more civic-minded and caring; they may build social responsibility into their company value and mission statements. (p. 28; Challenging)155. How is marketing being applied in the not-for-profit sector? Firms in the not-for-profit sector use marketing to enhance their images, to encourage donor marketing to attract memberships and donors, and to design social marketing campaigns to encourage specific causes. (p. 28; Moderate) Scenario Carol Veldt, owner of Seagull Terrace, watched her inv estment grow from a small, seaside motel to a thriving year-round resort in just a few years.Atop a bluff overlooking the Maine coast, Seagull Terrace had attracted thousands of visits during the summer months, but then faced a tremendous downturn in business during the winter months. â€Å"But, given the industry in the nearby towns, very little year-round competition, and our close proximity to Portland,† Carol added, â€Å"I couldn’t understand why seasonality had to hit Seagull Terrace so hard! † So Carol spent her first winter devising a new marketing plan. She put together a promotional package designed to attract business travelers year-round.Carol’s plan, then, involved a seasonal promotional gimmick—to be implemented from late winter to late spring—that would attract the large summer crowd. Her idea worked! During her second winter, Carol greeted numerous business travelers—both satisfied repeat guests as well as new guests w ho had been snagged by her promotional appeals. â€Å"We still have a long way to go,† Carol Veldt admitted. â€Å"Our delicatessen offers delicious entrees, but we’d like to expand that. We provide health club privileges off-site, but we’d like to eventually provide our own. These are goals I hope to achieve in a few years.Our first project, however, included a renovation of our guest rooms and I’m quite proud of the results. † Carol then added, â€Å"Actually there are so many possibilities! With an indoor pool area, I will eventually offer weekend get-aways throughout winter. †156. Based on the marketing process, what are Carol Veldt’s strengths? Carol was able to understand her customers’ needs and wants. She was able to deliver superior value through her marketing program, which created customer delight. (p. 6; Easy)157. What is included in the marketing offering at Seagull Terrace? Seagull Terrace provides activities and amenities that make a night’s stay more satisfying; these various activities and amenities are sought by two targeted groups— seasonal visitors and year-round business travelers. (p. 7; Moderate)158. How is Carol Veldt attempting to create brand experiences for her visitors? Carol is attempting to include numerous services and amenities for her visitors. Eventually, everything the visitors want or need will be offered at Seagull Terrace. (p. 7; Moderate)159. How has Carol Veldt taken on the role of marketing manager? Carol is attempting to find, attract, keep, and grow target customers by creating, delivering, and communicating superior customer value. (p. 8; Moderate)160. Define the target market at Seagull Terrace. Two types of guest are being lured: seasonal visitors during the summer and year-round business travelers. (p. 9; Easy)161. In what ways might Carol Veldt be implementing the product concept? Carol understands that guests will favor services that offer the most in quality and innovative features; Carol’s strategy currently focuses on making continuous improvements. (p. 10; Challenging)162. In what ways might Carol Veldt be implementing the selling concept? Carol understands that the success of Seagull Terrace, as she views it, requires a large-scale promotional effort. (p. 10; Challenging)163. How might the marketing mix at Seagull Terrace differ between its two target markets? Business travelers may be offered a discount business rate; obviously, the promotional tactic will differ for these guests. Summer guests may pay higher rates, but the beauty of Maine’s coast and the beach, as well as Seagull Terrace’s variety of services, will be the main attractions. (p. 3; Easy)164. How will Carol Veldt guarantee customer satisfaction? Carol will attempt to create services and amenities that exceed buyer expectations. (p. 14; Easy)165. Explain how Carol Veldt is engaging in partner relationship management. Explain how t his could be enhanced. Guests at Seagull Terrace currently receive health club privileges at a nearby health facility. Guests during the summer could receive sailboat rentals through such arrangements; year-round business travelers could be given meal discounts at local restaurants, dry cleaning services, and so forth. (p. 19; Challenging)